Jump to content

Wikipedia:Reference desk/Miscellaneous

From Wikipedia, the free encyclopedia

This is an old revision of this page, as edited by AlMac (talk | contribs) at 04:58, 3 October 2005 (→‎Programming in Fortran). The present address (URL) is a permanent link to this revision, which may differ significantly from the current revision.

Science Mathematics Computing/IT Humanities
Language Entertainment Miscellaneous Archives
How to ask a question
  • Search first. It's quicker, because you can find the answer in our online encyclopedia instead of waiting for a volunteer to respond. Search Wikipedia using the searchbox. A web search could help too. Common questions about Wikipedia itself, such as how to cite Wikipedia and who owns Wikipedia, are answered in Wikipedia:FAQ.
  • Sign your question. Type ~~~~ at its end.
  • Be specific. Explain your question in detail if necessary, addressing exactly what you'd like answered. For information that changes from country to country (or from state to state), such as legal, fiscal or institutional matters, please specify the jurisdiction you're interested in.
  • Include both a title and a question. The title (top box) should specify the topic of your question. The complete details should be in the bottom box.
  • Do your own homework. If you need help with a specific part or concept of your homework, feel free to ask, but please don't post entire homework questions and expect us to give you the answers.
  • Be patient. Questions are answered by other users, and a user who can answer may not be reading the page immediately. A complete answer to your question may be developed over a period of up to seven days.
  • Do not include your e-mail address. Questions aren't normally answered by e-mail. Be aware that the content on Wikipedia is extensively copied to many websites; making your e-mail address public here may make it very public throughout the Internet.
  • Edit your question for more discussion. Click the [edit] link on right side of its header line. Please do not start multiple sections about the same topic.
  • Archived questions If you cannot find your question on the reference desks, please see the Archives.
  • Unanswered questions If you find that your question has been archived before being answered, you may copy your question from the Archives into a new section on the reference desk.
  • Do not request medical or legal advice.
    Ask a doctor or lawyer instead.
After reading the above, you may
ask a new question by clicking here.

Your question will be added at the bottom of the page.
How to answer a question
  • Be thorough. Please provide as much of the answer as you are able to.
  • Be concise, not terse. Please write in a clear and easily understood manner. Keep your answer within the scope of the question as stated.
  • Link to articles which may have further information relevant to the question.
  • Be polite to users, especially ones new to Wikipedia. A little fun is fine, but don't be rude.
  • The reference desk is not a soapbox. Please avoid debating about politics, religion, or other sensitive issues.

NIT2WNIT

What does NIT2WNIT mean???

Uh...all I can come up with is "Need To Win it". Anyone else?--inks 02:11, 26 September 2005 (UTC)[reply]
It's "In it to win it" Proto t c 09:44, 26 September 2005 (UTC)[reply]

Help with translating this into 1337

Alright, I know this is a very odd question, but I would like help with obfuscating this sentence with 1337. The more obfuscated the better; the more symbols used the better:

Frequency estimation has fundamental significance in statistical signal processing.

Thanks for your help! --HappyCamper 02:56, 26 September 2005 (UTC)[reply]

i'll give it a try

ph|23/\|<\/\/1C 35+1^^/\+10h ]-[/\5+ ph|_||\||)|_|^^3|V+^|_ 51Oj|\|1ph1|</\|\|53 1n 5+/\+15+1</\|_ 51oj|\|/-\1 |o|2[]53551|\|0;!!!!!!!!!!!!!1111111oneoneone111!!! -Drdisque 05:53, 26 September 2005 (UTC)[reply]

Chargers' name

Are you asking about the San Diego Chargers? If you have a question that the article doesn't answer, you'll have to phrase your question in more of a complete sentence so that we understand what it is that you're looking for. Dismas|(talk) 03:31, 26 September 2005 (UTC)[reply]

The Chargers originated in Los Angeles in 1960. At that time, the Los Angeles Dodgers had a fan chant in which a recorded bugle sound played "Da da da DA da daaaaaa" and the crowd would respond with, "Charge!". The Chargers' name was meant to capitalize on the well-known cheer. User:Zoe|(talk) 04:23, 26 September 2005 (UTC)[reply]

What are all the factual inaccuracies in the book God's Debris by Scott Adams? I was trying to find a list off all the facts that he made up himself.

That would be Scott Adams, not Scott Adams. User:Zoe|(talk) 00:45, 29 September 2005 (UTC)[reply]

Geography Questions - Ethnicity

hello, we have a question on our geography revision...

Name ethnic groups which arrived in Brisbane, Queensland, Australia in the 1950's and 60's, another group which arrived in the 1970's & 80's and another group that arrived in the 90's and 2000's.

if you could help that would be great!! thanks!!!

165.228.131.11 04:26, 26 September 2005 (UTC)two geog students[reply]

  • I would try Greeks and Italians during the 1950's and 1960's, the Vietnamese for the 1970's and 1980's and the Chinese in the 1990s although there was an earlier influx of Chinese after the goldrush. Our Demographics of Australia article contains more information about Australian population trends. Please contact me if you need more info. Capitalistroadster 10:46, 26 September 2005 (UTC)[reply]

what age is a fetus considered a baby

legally, birth, morally, well thats up to you -Drdisque 05:55, 26 September 2005 (UTC)[reply]

It depends on whose doing the considering. This is one of the most controversial issues in politics, particularly American politics, because of its implications for the abortion debate. Go read that article as well as abortion debate to get a sense of the range of opinions. --Robert Merkel 06:27, 26 September 2005 (UTC)[reply]

Supreme Court

what are the names of the nine Judges in the Supreme Court?

See the article on United States Supreme Court and you'll see that your question isn't exactly a simple one at this time in history. Dismas|(talk) 04:45, 26 September 2005 (UTC)[reply]

Travel from Bahrain to Iran

Can you get to Iran by sea from Bahrain?

Principles

Stick to 'em! — ceejayoz .com 14:18, 2 October 2005 (UTC)[reply]

ERP configuration

what is meant by configuration of an ERP Application?

  • ERP is an abbreviation that can mean many things.

See if you can find a useful link in that article. - Mgm|(talk) 09:33, 26 September 2005 (UTC)[reply]

geography

plz tell me about geomorphology of ocean floor

Lloyds "Names"

Please will you tell me what is a "Lloyds Name" and what qualifies one to become a "Lloyds Name"?

  • Our article on Lloyd's of London will help you out. It states "For most of Lloyd's history, rich individuals ("Names") backed policies written at Lloyd's with all of their personal wealth (unlimited liability). Since 1994, Lloyd's has allowed corporate members into the market, with limited liability. The losses in the early 1990s devastated the finances of many Names (1,500 out of 34,000 Names declared bankruptcy) and scared away others. Today, Names provide only 20% of capacity at Lloyd's, with corporations accounting for the rest. No new Names with unlimited liability are admitted, and the importance of individual Names will continue to decline as they slowly withdraw or die off." Capitalistroadster 10:52, 26 September 2005 (UTC)[reply]

Health system in India

See Healthcare in India. olivier 06:35, 27 September 2005 (UTC)[reply]

Engrish

Rich successful China manufacturer electronic goods excellent, in this condition why instruction of manuals always confused utter English? Shantavira 12:23, 26 September 2005 (UTC)[reply]

  1. Translation is hard, particularly for linguistic nuances
  2. Good products with bad manuals sell better than bad products with good manuals — Lomn | Talk / RfC 12:49, 26 September 2005 (UTC)[reply]
Also, many companies (worldwide) neglect documentation as a time or cost saving measure, and because of the difficulty of retaining good technical authors. akaDruid 11:26, 28 September 2005 (UTC)[reply]

role of the male turtledove

Procreation? For some info, why don't you try the turtledove article? Dismas|(talk) 13:02, 26 September 2005 (UTC)[reply]

Rescue markings

In the recent hurricanes, rescue workers went house to house marking the buildings with a large cross, with numbers in the four quadrants made by the cross. I know that one quadrant contains the date, another contains a number indicating survivors, but don't know what the others are. What are these called? Is there a standard, or does each agency use a different system? Thanks!

Breslov Chasidus

Breslov Chassidus probably needs an article - [2], [3].

You would probably be interested in our Breslov (Hasidic dynasty) article.--Pharos 15:59, 26 September 2005 (UTC)[reply]

Thanks - should this be a redirect? Do we need another article?
OK, I've created a couple of redirects, which is easy to do. There's no need for two articles on the same topic of course.--Pharos 16:46, 26 September 2005 (UTC)[reply]

hurricane

Hurricane? Hurricane Katrina? Hurricane Rita?

Or in the Pacific, a Typhoon, Different names for the generic Tropical cyclone. So, what is your question? --hydnjo talk 06:42, 27 September 2005 (UTC)[reply]

prairie grass

Why were the plains states grasslands with few trees while other parts of the country was forested? Thanks.

--69.166.8.59 15:09, 26 September 2005 (UTC)[reply]

where can I find information about careers in the Army and RAF?

Which army and air force?

fake student

Hi. I read an article about a fabricated student who actually graduated and was enrolled to some school again and again. I don't remember the school name, or who this "person" was, do you? Punkmorten 19:17, 26 September 2005 (UTC)[reply]

The student was George P. Burdell and he attended the Georgia Institute of Technology -Drdisque 03:29, 27 September 2005 (UTC)[reply]

boxing terms: PTS

In boxing terms what doees PTS stand for and what kind of term is that? Heres an example: winky wright vs fernando vargas= W PTS rd 8 of 8(this is not actual info just an example)

Not an expert, but I'd guess PTS is a shorthand for a victory on points (by the decision of the ringside judges), rather than by a knockout (KO), disqualification (DQ) or technical knockout (TKO). The article on boxing has more detail on the significance of the various kinds of victories. ByeByeBaby 22:13, 26 September 2005 (UTC)[reply]

Hanseatic League, Papal decrees, Fish on fast days

Hi all you history buffs; This is a tough one . When I was in college a few millenium ago, I took a History of Religions course . In this course the professor told us that the Hanseatic trade league had persuaded( Bribed) one of the Popes to decree that on fast days meat was not allowed to be eaten and that fish should be put in its place. Fish was one of the main trading products of the league. We all know that fish are definetley part of the christian faith , ever since the loaves and fishes on the mount and Peters Gig as a Fisherman. My question to you is have their been any Papal or Church Bulls or decrees or treaties specifically made with the Hanseatic Trade League concerning fish trade and have their been any decrees by the Church requiring Christians to eat fish on Fast days? I have been trying to find an answer to this question for years and have only found one reference to Papal decree That christians should eat fish on Fridays and it had no specifific details. Thanks gang Dennis Nigrelli GOOD LUCK on this one:)

Well obviously (since everyone remembers this, right?) Catholics were forbidden to eat meat on Fridays until some time in the 1960s, and ate fish instead. And I think they still do that during Lent. But I don't know which popes decreed what. Michael Hardy 23:16, 26 September 2005 (UTC)[reply]
Pope Nicholas I (858-867) decreed that abstinence from flesh meat on Fridays was required. [5], but the tradition of abstaining from meat predates this (at least Tertullian) . There was never any requirement to eat fish - that's just urban legend stuff - really more anti-Catholicism than history. And the requirement certainly predated the Hanseatic League. The presently operative document is Pope Paul VI's Paenitemini (the Apostolic Constitution on Penance), published February 17, 1966. The operative Canon Law was Canon 1253: "The Episcopal Conference can determine more particular ways in which fasting and abstinence are to be observed. In place of abstinence or fasting it can substitute, in whole or in part, other forms of penance, especially works of charity and exercises of piety.” The effect of this was that the bishops of a country could decide what was required on Friday, and so the United States Conference of Catholic Bishops did....saying that prayer and penance could be substituted for abstention from meat on Fridays (of course, the requirement for a substitute act of piety is often neglected in the rush for the roast beef). Other country's decisions varied. this page is helpful - 00:04, 27 September 2005 (UTC)

Latinos on television

Sesame Workshop says: "Emilio Delgado (Luis) can easily claim one of the longest running parts for a Latino actor in a continuing series as "Luis" on Sesame Street, since the show's third season." That put his debut in 1971, or a 32-season role. Can anyone think of a longer running Latino television character in a non-Latin country? For that matter, can anyone think of a character in a Spanish/Mexican/South American-produced series that has appeared that long? -- user:zanimum

boat service from los cabos

can i take a boat or ferry from cabos san lucas mexico to acapulco mexico or other cities in mexico

thanks

La Paz, Baja California Sur, would be a more likely bet, for Topolobampo or Mazatlán. Hajor 00:43, 28 September 2005 (UTC)[reply]

Cattle & Irrigation

Is there a good way to keep cattle from breaking off irrigation sprinklers in a pasture?

The only thing I can think of is putting metal cages around the sprinklers and just dealing with the deflection caused by them. Also, an elevated ring around the sprinkler would also work.

Depending on your circumstances of course but it may be that electrical fencing would keep the critters away from easily damaged areas. --hydnjo talk 06:34, 27 September 2005 (UTC)[reply]

Basketball

Does Shaq play basketball because he chose basketball or because basketball chose him.

what is technology-formation?

What is the meaning of life?

I thought it was obvious enough! But maybe not... reproduce AND HAVE FUN!! That is if your fortunate enough to live in a free country

I looked at the Wikipedia article, but it offered no definitive answers. I want this question resolved before I expend any more energy on my high school education.

42? :)
On a personal opinion, there's no meaning. Why you think there should be one? And in this scenario, since there is no answer to "why?", we could pretty much just focus on finding out how. But that's just me... Kieff | Talk 11:36, 27 September 2005 (UTC)[reply]
It depends on what the meaning of 'is' is. Qaz 11:40, 27 September 2005 (UTC)[reply]
Guess what! You get to choose your own answer. Luckily, it's a multiple choice test and if you are truly blessed you can choose more than one answer. Most grown human beings try to do one or more of the following:
  1. acquiring as much as you can of
    1. knowledge
    2. wisdom
    3. wealth
    4. bodily pleasures
    5. power over other people
    6. piety or holiness points
  2. redistributing as much as you can of any of the above to other people
  3. attempting to contact or stay in contact with God
  4. helping as many people as possible to acquire some of the above
  5. building loving relationships with a limited number of people
  6. avoiding relationships with people and focusing on
    1. your inner life
    2. an imagined world
    3. changing the non-human environment or animals around you
  7. killing or injuring those who have offended you
  8. feeling sorry for yourself because you haven't been given enough of one of the above
  9. whining that "there's nothing to do here"

Of course, you can always toss in your cards and hope for another deal with another consciousness... Make a good choice. alteripse 12:16, 27 September 2005 (UTC)[reply]


What? Oh... I've been watching TV. But, I built a computer that could answer the question...

Bussiness proposal

Please help me to draft a business proposal for US based company

Um, you could start by reading our (relatively brief) article at business plan. Have you considered going to a library and borrowing a book on the topic? You might also benefit from some of the material on Wikibooks' School of Business, but I have no idea at what state that project is in.
However, given that you're presumably doing this to, ultimately, make money for yourself, if you want extensive assistance on preparing a business plan you will probably have to pay for that assistance. --Robert Merkel 14:32, 27 September 2005 (UTC)[reply]

herstory or history?

I would like to find out about herstory. They dont seem to teach it in schools. Can anyone help me?

Let me introduce you to Rocky and Bullwinkle and their way-back machine. Step in. We will set the dial to 1976. Ignore the bell bottoms, leisure suits, and the upcoming election between Ford and Carter. There is no internet but if you hang around on a college campus and look for the people who use the word womyn, they will be able to teach you some herstory. Once they became the prevailing viewpoint in academia, of course, they reverted to calling it history because the word never did derive from the masculine possessive pronoun and was one more perversion of the language for the sake of political correctness. Do we have an article on herstory? Since it turned blue, you can click and learn more. alteripse 14:43, 27 September 2005 (UTC)[reply]

Alteripse is being a bit curmudgeonly. If I may put what I think was his main point (aside from his distaste for language-twisting) a little bit more diplomatically: while feminist history is a valuable and (in my opinion) necessary view of our past, *any* one perspective on history is likely to provide an incomplete view. Variety is the spice of life! By the way, women's history and history of feminism may also be of interest to you. --Robert Merkel 14:49, 27 September 2005 (UTC)[reply]
Me a curmudgeon? I gave a concise accurate answer that included the historical, social, political, and linguistic dimensions in an entertaining, learner-friendly narrative. With a free wiki-link! What more could 'e ask for? alteripse 15:04, 27 September 2005 (UTC)[reply]
He could ask to see you "pull a rabbit out of this hat".... Thanks for the image of "Peabody's Improbable Herstory". <g> - Nunh-huh 00:27, 28 September 2005 (UTC)[reply]
Careful! No sexist pronouns. We don't know that 'es a he, now, do we? I think you need a refresher couse in language sensitivity. alteripse 00:38, 28 September 2005 (UTC)[reply]
There are no accidents. - Nunh-huh 02:46, 28 September 2005 (UTC)[reply]

what is nymphomax

Do you mean nymphomaniac? --Gareth Hughes 16:09, 27 September 2005 (UTC)[reply]

2001 film list

perhaps 2001 in film can help -Drdisque 17:57, 27 September 2005 (UTC)[reply]

Finertia

I've been to the site itself and all (it's a lifestyle community with an emphasis on healthy living), but still don't know what the name really mean?--221.219.138.61 19:08, 27 September 2005 (UTC)[reply]

So why don't you ask them? They would most likely know better than we do. Dismas|(talk) 20:20, 27 September 2005 (UTC)[reply]

What was the population of China in 1880?

This page suggests around 370,000,000 people. Shimgray | talk | 20:28, 27 September 2005 (UTC)[reply]


The Five People You meet in Heaven - is this book third person omniscient

Do your own homework. Read the book. Does the narrator say "I", "you", or "he"? Is the narrator fully aware of only himself or everything? — Lomn | Talk / RfC 19:53, 27 September 2005 (UTC)[reply]

Prime Minister

What is the last name of the prime minister that has ten letters in it.

Don't shout please. Now, your question is vague. What is the last name of the prime minister whose name has 10 letters? what is the last prime minister to have a last name of ten letters? Is it ten or any ten and up? What country's prime minister? Anyway, I'll guess that List of Prime Ministers of the United Kingdom can help you out. — Lomn | Talk / RfC 19:57, 27 September 2005 (UTC)[reply]
Interesting trivia: no UK PMs have had exactly ten letters in their last name, though a few have had ten-letter titles. — Lomn | Talk / RfC 20:04, 27 September 2005 (UTC)[reply]

Pokemon

Who 'invented' the Pokemon franchise? It's... Thelb4! 19:54, 27 September 2005 (UTC)[reply]

according to the Pokemon article you linked to, Satoshi Tajiri created the little deamons. Gentgeen 20:26, 27 September 2005 (UTC)[reply]

how many miles is it from new haven, connecticut to hartford, connecticut

Why not try mapquest.com or http://maps.google.com/ ? Dismas|(talk) 21:39, 27 September 2005 (UTC)[reply]
It's about 40 miles. -Nunh-huh 00:22, 28 September 2005 (UTC)[reply]

Music Sample

I've been listening to Sexy by the Black Eyed Peas for a while now, and I just can't place the music they 'borrowed' for it. It's the violin bit at the beginning, which pops up again throughout the song and at the end in reversed violin and guitar. I'd appreciate some help. -Flatluigi 21:33, 27 September 2005 (UTC)[reply]

  • Flatluigi,

The credits on Allmuisic.com for Elephunkcontains no credits for violin see [6]. However, this VH1 interview [7] from August 2003 states that Sergio Mendes played the piano on the track "Sexy" and the band got an orchestra in for "Where is the Love" so perhaps they got the violinist to play on the track. Sorry I can't help you more. Capitalistroadster 01:01, 28 September 2005 (UTC)[reply]

Now that I listen to it again, it is possible that it's a saxophone. Would I be breaking any copyright rules if I upload to Wikipedia said sample? -Flatluigi 01:13, 28 September 2005 (UTC)[reply]

  • If it's just a sample of the violin and if the sample is deleted once your question is answered, I don't think there'd be any law problems. - Mgm|(talk) 07:55, 28 September 2005 (UTC

Alright. Media:BEP_Sexy_Sample_1.ogg is the section I'm asking about. I'm currently uploading the other section with it. -Flatluigi 01:00, 29 September 2005 (UTC)[reply]

Media:BEP_Sexy_Sample_2.ogg is the second one. Thanks for your help. -Flatluigi 01:13, 29 September 2005 (UTC)[reply]

Punctuation

Articles printed in magazines end with a speclized symbol indicating that it is the end of that particular article. What do you call that symbol or puncutation mark?

You're probably referring to printer's dingbats, aka printer's ornaments. - Nunh-huh 00:19, 28 September 2005 (UTC)[reply]
See also Bullet (typography). Shantavira 17:30, 28 September 2005 (UTC)[reply]

Ostrich Meat

About how much calories and fat would about 1 ounce of Ostrich meat contain?

That would be hard to say since the amount of fat per lean meat could vary greatly. Dismas|(talk) 23:55, 27 September 2005 (UTC)[reply]
but it's still possible to say something about it. According to this page: [8], fat content varies from 1-4%, and there are about 110 kcal/100 grams. Taking the fattiest cut, the "oyster", as 4% fat, the content in 100 grams would be 29 grams of protein, 67 grams of moisture, 4 grams of fat, and (29 grams * 4.3 kcal/gram) + (4 grams *9.02 kcal/gram) = 160.8 kcal. The percentage of calories from fat would be 36.1/160.8 = 22%. 100 grams is about 3.5 ounces. So one ounce of ostrich "oyster" meat would contain 8 grams of protein, 19 grams of moisture, 1 gram of fat, and 46 kcals. - Nunh-huh 00:17, 28 September 2005 (UTC)[reply]

King Arthur

... is this way, past the toilets and the second on your left. --fvw* 00:09, 28 September 2005 (UTC)[reply]

ambiguous comments make it hard to.......just whta the speaker meant

…Understand? I don't know, your question is kind of vague. Garrett Albright 12:36, 28 September 2005 (UTC)[reply]
Fvw (above) is inviting you to click on his link. Shantavira 17:33, 28 September 2005 (UTC)[reply]

what is an axe murderer?

Western Movie Theme Songs

Talking in freefall

I had several questions about skydiving. Is it possible for skydivers to talk during freefall, and can they even breath before the open their parachute?

yes and yes, though each bears a note. Talking would be better stated as shouting during skydiving due to ambient wind noise. Unassisted breathing may not be possible under some HALO/HAHO jumps due to the altitude at jump; however, this is in no way a result of the jump itself. — Lomn | Talk / RfC 02:51, 28 September 2005 (UTC)[reply]

Dentists

If you clean your teeth real well, is there really a need for Dentists? Why can't regular doctors learn oral surgery?

Cleaning your teeth real well doesn't eliminate dental disease, so yes, there is really a need for dentists. And many oral surgeons are "regular doctors", if by that you mean they have an "M.D." degree. - Nunh-huh 03:57, 28 September 2005 (UTC)[reply]

Teeth cleaning may reduce frequency of caries but does not prevent all dental problems and their are many other things that can happen to your teeth. The separation of dentistry from surgery and medical practice is partly a matter of historical contingency. In the Middle Ages physicians were distinct from barber-surgeons who also performed surgical and dental procedures. Between 1300 and 1600, haircutting and dentistry became separated from surgery because people tended to specialize in one of the activities. After about 1800, surgery became a part of medical practice. The basic science training of physicians and dentists is similar but has become part of training only in the last century (the split predates it). The other main historical reason is the dentistry is also a craft of manual skills relatively easy to separate from those needed for surgery or medical practice. A lot of dental school is learning the manual craft and it would add a couple of years to medical school to teach physicians to do this. Especially for manual skills like surgery and dentistry, you want someone to do it a lot to be good at it. Finally, as Nunh-hunh says, there are oral surgeons who got there through dental school (DDS) and those who got there through medical school (MD). alteripse 09:19, 28 September 2005 (UTC)[reply]

Worldwide Cumulative GPA Results

Does anyone know where I can find statistics on average worldwide cumulative GPA results and also by country/area? Thanks for any help with this.

GPA is used at University level in New Zealand. I know, I've been at Uni far too long. I suspect it would be pretty trivial to assign a numerical value to a grade and count them up anyway - which is a good thing. It would allow me to compare country X which goes by A, B- etc., and country Y, which goes by "Exellent", "Merit", etc. If I can convert them to numbers, I can run Stats™ on them. Not the most elegant of examples I know, but the point is made.--inks 09:14, 28 September 2005 (UTC)[reply]

IB Schools in Asia

Does anyone know who were the first 2 IB schools in Asia? Thanks for this.

Chad Chunning

What did Chad Chunning do from quitting nirvana until joining his new band?

Bob Jones

Can anyone tell me anything about Bob Jones

Capitalistroadster 05:33, 28 September 2005 (UTC)[reply]

charmane star

what movie did charmane star do a double penetration scene in ? what is the movies title ? what year was the adult movie released and by what adult film company was it distibuted by ? at your website a page on charmane star mentions trivia on her that mentions a double penetration scene but does not name the movie she did it in. --will

Will, you might have better luck posting this question on the article's talk page. You might also like to try perusing her official website (which is linked from our Charmane Star article) to see if it has more details. If that fails, you could even try contacting her representatives through the website to ask. I suspect that they'd be more than happy to sell you a copy. --Robert Merkel 04:43, 29 September 2005 (UTC) (Always happy to help here at the reference desk, no matter how unusual the request.)[reply]

How to write a monologue

I was just wondering, what are the basic parts when it comes to writing a monologue. We have to do a radio monologue and it's really annoying me.

I hope you can help. Thanks

is this monologue for english?? i have to do one too!!! i think you go to the same school as me...it's possible... i'm guessing if you are doing the same one as me, you have to present it on a tape, right? well anyhow the one i have to do, (the positioned radio monologue), you have to pick two speakers, and have them say their opinion in their discourse... meaning if you got like a yobbo as one, they must talk like a yobbo and use words a yobbo says... and the other speaker has their discourse too... for example one person talks about going to uni after high school and the other talks about doing an apprentinceship...
Kattus 11:37, 28 September 2005 (UTC)[reply]


A monologue is a speech. You have a message and an audience. Your job is to communicate the message in words so that it will understood by the audience and elicit the response you want. Are you having trouble deciding what the message should be, or what response you want from the audience, or are you having trouble with deciding how best to get the message across? alteripse 11:41, 28 September 2005 (UTC)[reply]

skydiving

What's the procedure for learning to skydive?

Most people join a sky diving club at the local airport or join the paratroopers. Trial and error is not a recommended method. alteripse 11:41, 28 September 2005 (UTC)[reply]

Just throw yourself at the ground, and miss. --Douglas Adams 13:31, 28 September 2005 (UTC)[reply]
Which describes flying, or putting yourself into orbit, not skydiving :) Generally, the club will familiarise you with equipment and procedures, before progressing to tandem, and finally solo jumps. You usually "pack" your own parachute.--inks 20:16, 28 September 2005 (UTC)[reply]
Skydiving is just flying in a strictly downward fashion. :) — Lomn | Talk / RfC 20:26, 28 September 2005 (UTC)[reply]

What does the word erradicate mean?

Nothing, but eradicate means to totally remove, literally to rip out by the root. alteripse 11:41, 28 September 2005 (UTC)[reply]

how come feburary only has....

how come feburary only has 28 or 29 (leap year) days when every other month has 30 or 31?? why didn't they divide all the months up evenly???

Our article on February has an excellent link to your exact question being answered on The Straight Dope. But answering your second question: because the division of the months was a long, stupid and biased process. I guess we could easily split the months evenly today, but the social inertia that'd require to break would be way too big to consider "convenient". While we're at that, see also: dozenalism Kieff | Talk 12:06, 28 September 2005 (UTC)[reply]

About this part in Half-Life²...

Before HL2 came out I watched a few videos, one of them depicted this part of the game (... with Barney and some resistence folks, then two striders came in.) The thing is, I got HL2 and finished it a couple of weeks ago and never saw that part of the game.

So I was wondering, did that part get cut off the final version, or was it made just for demonstration, or I just did something wrong while I was playing it? Kieff | Talk 11:55, 28 September 2005 (UTC)[reply]

  • Probably just got cut from the final version. Lots of stuff that was in HL2 previews got cut due to AI/playability problems (e.g., the famous blue tentacle-thing) or changes in the plot (e.g., the whole icebreaker section). If you read Half-Life: Raising the Bar, you'll see tons of stuff that never made the final game. I suspect at least some of it will appear as expansion packs, etc. -- Bob Mellish 17:47, 28 September 2005 (UTC)[reply]

city area of Pyongyang

did you have a question??

We have an article on Pyongyang, the capital of the Democratic Peoples' Republic of Korea (North Korea). Garrett Albright 12:31, 28 September 2005 (UTC)[reply]

bussiness in pusan

We have an article on Busan/Pusan. There are probably many businesses in this large South Korean city. Garrett Albright 12:38, 28 September 2005 (UTC)[reply]

hockey

Hello,

Who was the first Swedish born player inducted in the Hockey Hall of fame?

Thanks for the help!

i did a google search with the keywords 'first swedish hockey player hall of fame' and this website seems to tell me that Borje Salming was the first Swedish player inducted in the Hockey Hall of Fame... hope this helps you... Kattus 12:25, 28 September 2005 (UTC)[reply]

how does a human being have the XXXY chromosome?

my biology teacher told us that it is possible (but rare) to have XXXY as your chromosomes instead of the usual XY or XX... is this true and how is that possible? does the person have any genetic complications because of this and what are they then..because the 'XX' part is female and 'XY' is male so wouldnt they be male and female at the same time???

can someone explain it or give a website that does? thanks! Kattus 12:13, 28 September 2005 (UTC)[reply]

Please see Chromosome#Chromosomal aberrations, although your teacher was probably talking about Klinefelter's syndrome (which is XXY, not XXXY). -- Rick Block (talk) 14:18, 28 September 2005 (UTC)[reply]

How does it happen? We think that at an early stage of zygote development, when chromosomes duplicate and separate into 2 cells, the separation failed to occur, leaving the surviving cells with one or more extra chromosomes. This is called nondisjunction. The reason the effects are so minimal for the people with multiple X (compared with having multiple copies of chromosome 21 or 1, for example), is that the second (and any excess beyond 2) X chromosome in mammals is mostly inactivated (Lyonization) in most cells most of life and therefore causes relatively little trouble. Whether one is male depends (over 99% of the time) on whether one has a functioning Y chromosome. The number of X chromosomes does not matter: both males and females have X chromosomes. You can have 1 X or 5 X chromosomes and you will be female as long as you do not have a functioning Y. See sex differentiation for more details. alteripse 17:58, 28 September 2005 (UTC)[reply]

Hockey #2

Hello!

Who is the first Team to have won the Stanley cup?

again i googled and wikipedia seems to have an article on the Stanley Cup.... so there you go! go down to 'Early Years', the last sentence on the first paragraph in that section should tell you the first team to win...

Kattus 12:30, 28 September 2005 (UTC)[reply]

how do you... wikipedia skills

i would like to know how you make a website link say a keyword so that when i post something up i dont have long website addresses (for example, when i answered the hockey question)... and how do you link the wikipedia articles so that again a keyword is highlight in blue and underline... like everyone else does... could you help? that'd be great thanks! Kattus 12:37, 28 September 2005 (UTC)[reply]

See Wikipedia:How to edit a page for your first question. As for the second question, a link will be blue if a linked article exists, and red if it doesn't exist. For example, we probably have an article for banana, but I doubt we have an article for xczv.nm,weroiu because I typed that by randomly smashing on my keyboard. Please use the Wikipedia:Help Desk for other questions along these lines. (See? I don't bite all newbies…) Garrett Albright 12:46, 28 September 2005 (UTC)[reply]

so does it automaticly links to the article?? or do i have to link it myself - but how?? Kattus 12:50, 28 September 2005 (UTC)[reply]

[[banana]] comes out as banana. For the most part, the article text stays the same as you wrote it when you press submit (of course it's translated into html when you view an article in the normal way), so you can see exactly how people do things when you edit an article/section. Frencheigh 12:55, 28 September 2005 (UTC)[reply]

so for example in the Hockey question above where i wrote 'i did a google search with the keywords 'first swedish hockey player hall of fame' and this website..." how do i make the word 'website' actually be a link to the website that is beside it... id love to know how to do that (if its possible) Kattus 13:00, 28 September 2005 (UTC)[reply]

You are looking for piping. To pipe a wikilink, you would use something like [[Banana|test1]], which looks "test1" but links to Banana. URL's (external links) can be piped too, but generally all external links go down the bottom in the "External links" section. Wikipedia:External link has more info, including how to pipe extenral links, which is different to internal wikilinks. (An example might be [http://www.banana.com test 2], which looks like: test 2. --Commander Keane 13:12, 28 September 2005 (UTC

well i know what you're talking bout now cos i read the wikipedia editing help and i figured out the keyword thing..you know how you type [ and then the website and then two spaces and then keyword and then close it with ] but yeah thanks for all your help everyone!! i really appreciate it! Kattus 13:20, 28 September 2005 (UTC)[reply]

Two sapces? One space works, is two good etiquette or something? --Commander Keane 16:20, 28 September 2005 (UTC)[reply]

meh!! thanks anyway! Kattus 22:04, 28 September 2005 (UTC)[reply]

Re: Wikipedia article: Uniformed services of the United States - more than 7?

Besides the seven services listed, why aren't the many other uniformed services of the United States listed? For example, within the United States Department of the Interior there are uniformed members of the National Park Service, Bureau of Land Management, and U.S. Fish and Wildlife Service. In the United States Department of Agriculture there are uniformed members of the United States Forest Service (several different types of Park Rangers, for instance). Perhaps other operating units of the DOI and USDA also have uniformed members. Perhaps other U.S. Cabinet-level departments have uniformed members as well. Why are none of these uniformed services of the United States listed? Is it simply because they are not military(naval)-ranked like the NOAA Commissioned Corps or the Public Health Service Commissioned Corps? Is it because they don't operate under military rules? (If so, shouldn't the title "Uniformed services of the United States" be qualified by some statement that indicates this limitation to the definition?) Or is there some other distinction? Courtneymitchell 13:49, 28 September 2005 (UTC)[reply]

And don't forget the uniformed service at McDonald's. --Gareth Hughes 14:19, 28 September 2005 (UTC)[reply]
The very first sentence in Uniformed services of the United States includes a link to uniformed services, which makes it clear that "uniformed services" does not just mean "any government employee who wears a uniform." What more do you want? Also, for future reference, issues you have with a specific article are usually better handled on the talk page for that article rather than here. Chuck 20:27, 28 September 2005 (UTC)[reply]
Sorry, that was an overly snide response. Let me rephrase it. The uniformed services article notes, "Uniformed Services refers to bodies of people in the employ of a state that are not employed on standard terms of contract, but have some element of additional discipline, and wear a uniform." (emphasis mine) So it's not just any government agency whose employees wear a uniform. If you think it would be helpful to include this qualification within the Uniformed services of the United States article too, feel free to go ahead and add it there. Chuck 22:31, 28 September 2005 (UTC)[reply]
The definition comes direct from U.S. law, specifically Title 10 of the U.S. Code. I will update the article to state this. Isomorphic 06:34, 29 September 2005 (UTC)[reply]

Thanks, I'm new to this process and appreciate the kindness. What a remarkable phenomenon is Wikipedia! I'm still interested in the other U.S. services that use uniforms such as the U. S. Park Service Park Rangers, the United States Park Police, USDA Forest Service rangers, Game Wardens, the United States Border Patrol, etc., their corresponding pay rates and rankings. The seven military services articles were very satisfying in scope and thoroughness. I'm still looking for something comparable for the other uniformed services, some of whom have extensive military training, carry weapons and put their life on the line every day.

Also of interest is the appearance of CIA operatives in Naval uniforms within the popular culture in such movies as "The Hunt for Red October" and "Clear and Present Danger". Is this a real practice and does it come from a reason to protect them against being captured as spies in a similar manner that was given for uniforming a non-military service such as the Public Health Service Commissioned Corps (in Wikipedia's ariticle Uniformed services of the United States)? Courtneymitchell 17:15, 30 September 2005 (UTC)[reply]

For another example of "agencies whose employees wear uniforms, but are not technically one of the uniformed services," don't forget what may well be the most visible of those: the United States Postal Service. Chuck 18:34, 30 September 2005 (UTC)[reply]

how do you cure ringworm?

see Ringworm#Treatment_and_prevention

ROOT FINDING METHODS

Is bisection or newton raphson faster? it's bisection, right? less iterations?

Depends on the situation. When Newton-Raphson works correctly, the convergance is quadratic, so the number of correct digits is roughly doubled each iteration. For bisection, the error is cut in half each iteration (so there's about 3 iterations to a decimal digit). There are cases where Newton's method simply doesn't converge, (a zero on an inflection point can do this, it keeps overshooting back and forth), but the bisection method doesn't suffer from this. Frencheigh 16:14, 28 September 2005 (UTC)[reply]
If Newton-Raphson was never faster than the Bisection method then we wouldn't hear about it, why bother with something that is slower and doesn't work sometimes? So, Newton-Raphson is probably faster than bisection most of the time (particularly since it uses extra information, the derivative). I could be wrong, but this is what makes sense to me.--Commander Keane 16:27, 28 September 2005 (UTC)[reply]
In practice, it is sometimes a good idea to use both. First use bisection to get a starting point for which Newton-Raphson is likely to converge, then use Newton-Raphson to achieve desired accuracy. If Newton-Raphson fails, improve the bisection estimate further and iterate. Fredrik | talk 17:20, 28 September 2005 (UTC)[reply]
It's also worth pointing out that slower algorithms are frequently taught first due to complexity. For example, virtually every Computer Science student is familiar with bubble sort even though merge sort or quick sort is preferable for virtually any large set. Consequently, "I've heard of it; therefore it's useful" isn't an optimum strategy. — Lomn | Talk / RfC 20:31, 28 September 2005 (UTC)[reply]

We have an article on root-finding algorithms. Gdr 21:11, 30 September 2005 (UTC)[reply]

New Orleans Levy Funding

What happened to the projects to update the levy system in New Orleans from when Clinton started funding the project to it's breakdown?--66.145.213.254 16:59, 28 September 2005 (UTC)[reply]

house painting after World War I

Without a question, all I can tell you is that they used paint. — ceejayoz .com 14:07, 2 October 2005 (UTC)[reply]

Chairman, Joint Chiefs

Can the Chairman of the Joint Chiefs of Staff stay in the military when thier term is over? How is the Chairman Chosen?

Our article at Joint Chiefs of Staff answers this: The chairman is nominated by the President and is at all times a member of the military, though detached from the regular chain of command. Because the Joint Chiefs are the highest-ranking members of their respective branches, they tend to end their terms on the collective council upon retirement. — Lomn | Talk / RfC 02:20, 29 September 2005 (UTC)[reply]

amusemn

amusement rides

What exactly is your question? Bart133 (t) 23:33, 28 September 2005 (UTC)[reply]

What do you call the printed brochure at a funeral?

I'm not asking about the eulogy, or the obituary, or an elegy--but about the printed thingy with a poem or prayer, the brief biography of the person, perhaps a photo--etc.

In Britain, it's known as an order of service; programme sounds far too much like something printed for an etertainment or sporting event. --Gareth Hughes 18:31, 28 September 2005 (UTC)[reply]
Are you talking about the cards that are often printed for funerals with a picture, possibly of Jesus Christ or some depiction of a saint, on the front and the person's name on the back with their date of birth and death? The back also sometimes has a small prayer or biblical passage printed on it. In the United States, I've often heard these being referred to as "prayer cards". I don't know if that's the official name for them though. Dismas|(talk) 23:26, 28 September 2005 (UTC)[reply]
That's unlike the order of service that Gareth mentioned. For a funeral, it generally list the name of the deceased person (so you know you're at the right one), gives the order of the service (listing hymns, readings, communion, etc.). Sometimes it has the words for hymns, and sometimes has a photo or two of the deceased person, details of any post service get together, and perhaps details of a charity to which you're invited to donate. -- Finlay McWalter | Talk 23:34, 28 September 2005 (UTC)[reply]

cedent

Hello:

What is a "cedent"? The term is used often in the insurance industry.

Please advise.

Thanks!

David

It is probably a person who cedes something. This is not a good way to use the English language. --Gareth Hughes 18:36, 28 September 2005 (UTC)[reply]
I guess they could have called them the cedar. DJ Clayworth 19:57, 28 September 2005 (UTC)[reply]
Or a seder. User:Zoe|(talk) 00:52, 29 September 2005 (UTC)[reply]

Star and Moon Atoll

I have an aerial view of two atoll's or islands next to each other shaped like a star and a crescent moon. There are large boats in the water around them, but I cannot find anywhere on the web where these atolls or islands are located. They look just like the Star and Crescent of the Red Crescent organization. I would like to know where they are located. --SG

I have seen pictures (I believe in Smithsonian Magazine) of manufactured coastal communities (in one or more of the oil-producing Arab states) where soil and sand have been moved in massive ways to form luxury waterfront properties that, when viewed from the air, show images such as you mention. The picture I recall was of such a development in the shape of a palm tree. Courtneymitchell 21:47, 30 September 2005 (UTC)[reply]

2 sticks butter are how much in the metric system?

Hello!

I have a wonderful recipe for even more wonderful "chunky chocolate cookies", which I found in a cookie bakery in New York City. But they gave me a recipe which is aimed more to the U.S. public, because there is no single indication of ingredients made in the metric system.

But that's fine; I don't think that teaspoons look in the U.S. that much different than in Europe and I can calculate how much 350° F are (or 300 when I use the convection oven...), but how much are two sticks (unsalted...) butter (United States) when indicated in a baking recipe?

  • Fourteen milliheartattacks. Seriously, though, butter is sold in the US in 1-pound packages of four quarter-pound sticks. One stick is 4 ounces, which is equivalent to a half a US cup. Cup indicates that there is only a 13-mL difference between US and metric cups, so you're probably safe using a metric cup of buttery goodness to make your cookies. android79 19:59, 28 September 2005 (UTC)[reply]
    • Whoops. Fluid ounces measure volume, while regular ounces measure weight. A stick of butter is four ounces in weight. To figure out the volume, you would need to know the density, which I don't. Superm401 | Talk 14:08, 29 September 2005 (UTC)[reply]
      • By the way, note that Europe doesn't generally use "cups" as a measuring device - I know cooks who've gone to the effort of ordering measuring cups all the way from the US in order to have to stop converting recipies all the time. Shimgray | talk | 15:01, 29 September 2005 (UTC)[reply]
  • Thanks a lot!! Hmmm, yummy! Dani_skr, 20:05, 28 Sep 2005 (UTC)
  • Well, I wonder how those cookies turned out, then... android79 15:05, 29 September 2005 (UTC)[reply]

methamphetaime/recovery symptoms

Good restaurant

What's a good restaurant in midtown, Atlanta, Georgia? Just looking to take some friends to dinner there, less than $20.

for visitors, I frequently recommend the Park Tavern (500 10th St NE, Atlanta, 30309). If you like sushi, Ru San (1529 Piedmont Ave NE, Atlanta, 30324) is very good and has great prices, If you require a place in the hart of midtown, I'd suggest The Vortex (fancy burgers) (878 Peachtree St NE # 4, Atlanta, 30309) or Joe's On Juniper (1049 Juniper St NE, Atlanta, 30309) (most of the waitstaff is pretty gay there if you don't like that sort of thing, but midtown as a whole is rather gay). -Drdisque 04:36, 29 September 2005 (UTC)[reply]

Penis enlargement

Do any of those penis enlargement pills actually work, or are they all snake oil?

Nope, snake oil. Penn & Teller devoted an episode of their television show Bullshit! to this in fact.
Wikipedia has an article on penis enlargement, though I would add that perspicacious prospective partners are more attracted to a pleasing personality than a pendulous penis. Shantavira 08:16, 29 September 2005 (UTC)[reply]

Vitamin C

IS it possible to overdose on vitamin C?

It is possible to overdose on any substance. Paracelsus, considered the father of modern toxicology, made a statement that is often paraphrased as "the dose makes the poison." Meaning, any substance is poisonous if you ingest enough of it; conversely, any substance is non-toxic in a sufficiently small amount. This MSDS for ascorbic acid (vitamin C) cites an LD50 of 11900 mg/kg in rats, and 3367 mg/kg in mice. Even if we use the lower number for mice and apply it to humans, it would take a few hundred grams of vitamin C (remember that vitamin C tablets are usually measured in milligrams) to be fatal. Chuck 20:15, 28 September 2005 (UTC)[reply]

Six Flags, atlanta, ga

Has anyone ever died at six flags in atlanta?

Searching through RideAccidents.com, which bills itself as "the world's single most comprehensive, detailed, updated, accurate, and complete source of amusement ride accident reports and related news," I found one report of a death: an employee was killed on May 26, 2002, when he was in a restricted area beneath the track of Batman The Ride, and was struck by the leg of a passenger. Chuck 22:10, 28 September 2005 (UTC)[reply]

Child porn

If you're under 18, and post pictures of yourself online, will you get arrested for chid sexual abuse? I need an answer NOW

  • Probably depends on which state you're in. In any case, if you're talking about nude or suggestive pictures, I'd advise against it regardless of the legality of it. On the Internet, everything lasts forever. android79 20:01, 28 September 2005 (UTC)[reply]
    • Can you 'abuse' yourself? Dubious legal argument at best. Recently, 2 underage girls in NZ presented themselves at a "gentleman's club", claiming to be over 18, and saw a few "clients" each. IIRC Neither were prosecuted. Having said that, some places still make it illegal for you to kill yourself. I would go with what Android79 suggested.--inks 20:08, 28 September 2005 (UTC)[reply]
Teen girl charged with posting nude photos on Internet "charged with sexual abuse of children, possession of child pornography and dissemination of child pornography." Frencheigh 23:07, 28 September 2005 (UTC)[reply]
American law enforcement just gets wackier all the time. (I am American). How did the case turn out?Nelson Ricardo 00:32, 29 September 2005 (UTC)[reply]

ME

What are the top 5 mechancal engineering schools in the nation?

That depends a whole lot on your metric. For starters, what nation? Assuming we can get a definitive answer there, do you care about just undergraduate, just graduate, or both? Do you want opportunities for undergraduate research at that campus? If graduate, masters vs doctorate, and what particular area? The list builds. Then we get into things like the realization that you won't (hopefully) spend all your time in class, so what about the campus-wide aspects, distance from home, cost, etc, etc? Point is, there are no definitive rankings, and any rankings that purport themselves definitive (c.f. U.S. News) won't be tailored to your specific goals.
However, to put in my 2 (useful) cents, you will get out of college what you put in. For undergraduate in particular, everybody's curriculum will be comparable. I'd look at things like campus life and industry affiliation (you do want a real job eventually, right?) over whether or not a prof has a Nobel Prize. Also consider things like school and class size: you may get a better education at a small school with generic profs who work regularly with students than at a big name school with lots of research where teaching is delegated to grad students. — Lomn | Talk / RfC 20:42, 28 September 2005 (UTC)[reply]

diet coke

Is aspartame bad for you?

See Aspartame#Health effects controversy. Chuck 21:09, 28 September 2005 (UTC)[reply]

toilet bowl cleaner

OK, I've got a crazy idea. You know those toilet bowl tablets? They're bleach tablets, same thing you put in a swimming pool, ie calcium hypochlorite. Can I save some cash and just use the chlorine tablets for my pool in my toilet bowl, or will something go horribly wrong?

yes, but they will possibly overchlorinate your toilet water and make your bathroom smell like a poolhouse. it could also corrode any metal in your toilet, including the handle and flapper assembly.

sanitizing tap water

IF you don't have iodine, can you sanitize water with a couple of drops of bleach?

Apparently you can as a last resort: this page from the CDC contains instructions in the "Water" section. Do keep in mind that too little bleach won't sanitize the water, and too much can cause personal injury. Please see Wikipedia's medical disclaimer. — File:Ontario trillium sig.pngmendel 01:18, 29 September 2005 (UTC)[reply]

What's a good stock broker

what's a good stock broker that allows otc bb trades and is less than 10 bucks a trade?

White noise

Do noise cancelling headphones actually work?

Yes. active noise cancelling headphones do actually work. But of course, you don't get 100% noise cancellation - monotonous sounds (like car or aircraft engines) get cancelled out quite well, but human speech and music doesn't. My noise cancellation headphones are awesome when flying (as a passenger), and pilots of light aircraft use them too. --inks 20:24, 28 September 2005 (UTC)[reply]
It's just about worth noting that noise cancelation uses reversed phase sound waves, and not white noise, which would merely mask the sounds. Sadly my bose headphones a) are sufficiently shielded to resist burbling in time to mobile phone polling signals; and there's some sort of spring mechanism in the headband which has started chattering away as I move my head. And yes, I'm wasting time whinging about it on wikipedia rather than boxing them up and sending them back. Doh. --Tagishsimon (talk)
Apparently they work better on low frequencies (train noise etc) -- Dan's Data has a short article about them here (scroll down to second section). Ojw 21:56, 30 September 2005 (UTC)[reply]

tesla coils

is there a max # of volts the human body can take (not talking about amps, i know we can only take a little bit of amperage) but is there a max voltage the human body can take?

If there are no amps then your body can take any number of volts. You may, in high school, have done the experiment where you stand on something insulated and then touch a Van de Graff generator, and your hair stands on end. That charges you to millions of volts, but because there is no current it isn't dangerous. DJ Clayworth 21:08, 28 September 2005 (UTC)[reply]
The ability of the human body to resisr the effects of electric shock varies enormously, and depends on various factors such as general fitness, sweatiness, and type of clothing worn. A recent news item reported someone who was killed while changing a domestic light bulb; on the other hand some people have survived multiple lightning strikes. Shantavira 08:28, 29 September 2005 (UTC)[reply]
Also, the time of exposure to the current is important. Our article on electric shock gives a better explanation on this subject. ☢ Kieff | Talk 09:45, 29 September 2005 (UTC)[reply]

deliverence

Who was the actor who played the banjo boy in deliverence? is he still alive today and if so does he act?

Here is the Internet Movie Database page about Deliverance (1972), which has a complete cast list. If you don't find it there I suggest asking at that site, because they have a much more movie-focussed readership. DJ Clayworth 21:10, 28 September 2005 (UTC)[reply]
We have an article on Deliverance as well as Billy Redden, the actor who played "Lonny" the banjo player. Redden is alive and was recently in the movie Big Fish although he is not an actor by profession. Dismas|(talk) 21:41, 28 September 2005 (UTC)[reply]

contact lenses

are the night and day 30 day contacts safe? i know they're fda approved, but leaving something in your eye for 30 days, can't that cause infection?

I would suppose that if you don't care for your eyes then yes, they could cause infection. Although if the FDA approved them, I would think that they believe them to be safe. Please consult a doctor if you have any problems with your eyes, and follow the directions on the packaging of any medical products. Dismas|(talk) 21:52, 28 September 2005 (UTC)[reply]

fish poisoning

how many ppl die from that fish that's poisonous but ppl still eat it?

From the Fugu article: "Some sources claim that about 100 people die each year from fugu poisoning, while others sources say only 10 to 20 per year, and still others state only 1 person dies each year from fugu. This reported variation may be the result of different sequences of years being studied..." Chuck 21:13, 28 September 2005 (UTC)[reply]

dividends

HAVE THE STOCK HOLDERS RECIEVED THEIR DIVIDENDS FOR THE THIRD QUARTER OF 2004? OR WAS THERE BACKUP WITHHOLDING?

Which stock? Stocks don't all issue their dividends on the same day. Chuck 20:54, 28 September 2005 (UTC)[reply]

do you know the muffin man?

the muffin man? the muffin man?

See Shrek. DJ Clayworth 20:56, 28 September 2005 (UTC)[reply]

We have a number of articles on Nursery rhymes but not that particular one it seems. Capitalistroadster 04:49, 29 September 2005 (UTC)[reply]
Sure, he hangs out on Drury Lane. Shantavira 08:31, 29 September 2005 (UTC)[reply]

lord forest

is there a movie with a character called lord forest?

Using IMDB's character name search returned A Royal Demand. Chuck 21:21, 28 September 2005 (UTC)[reply]

wireless internet

what's the max distance for a wifi conncetion?

In the Wi-Fi shootout competition at this year's DEF CON, the winning team managed to sustain an 11Mbps connection over a distance of 125 miles, using one 12 foot dish antena and one 10 foot dish. (Slashdot report) Of course, in any normal situation, the range will be far less: our article on WiFi suggests 45-90m (150-300 feet) as typical for access points sold for home use - the actual range is dependent on many variables, for example physical features such as walls absorbing the signal and reducing range. I would say the best way to find out what range you'll get in a specific location with specific kit is to test it. -- AJR 22:29, 28 September 2005 (UTC)[reply]

longhorn

when's windows longhorn coming out?

Why not read our article on Windows Longhorn and find that out plus a whole lot more? Dismas|(talk) 21:00, 28 September 2005 (UTC)[reply]
Right after Duke Nukem Forever, the end of the gulf war, and the Hades ice rink. Ojw 21:59, 30 September 2005 (UTC)[reply]

tallulah falls, ga

does anyone know if the river there is open to kayaking?

sustainable height

what's the highest distance someone can fall and still live (comatose = alive, for this question)

There are quite a few factors to take into account there. Is the person landing on their feet or their head? What are they falling onto, grass or concrete? etc. Dismas|(talk) 20:56, 28 September 2005 (UTC)[reply]
Do they have a parchute? Were they concious at the start of the fall? Thryduulf 21:13, 28 September 2005 (UTC)[reply]
In certain circumstances the answer is infinte (in theory) - if they are in free-fall and have apropriate protection from space (e.g. a space-suit or being inside a spacecraft that is also in free-fall) then the whole time they spend in orbit they spend falling.
The longest time in space aboard a single flight is 437-days, 17-hours, 58-minutes, 04 seconds (onboard Mir by Valeri Polyakov). If we assume for simplicity that exactly 437 days of that was spent in orbit, and that mir had a 90-minute orbit (it was 89.1 minutes) and that each orbit was exactly 25,000 miles.
Our article on Dr Polyakov states he is still alive, meaning he must have survived the fall of over 174 million miles. Thryduulf 21:41, 28 September 2005 (UTC)[reply]
If we avoid being extra-terrestrial Free-fall metions some plummeters. MeltBanana 23:15, 28 September 2005 (UTC)[reply]
Depends entirely what you happen to land on. A number of people have survived a fall from a commercial airliner that broke up at cruising altitude. Our article on accidents and incidents in aviation mentions "a 17-year old girl who survives a fall from 2 miles". Shantavira 08:41, 29 September 2005 (UTC)[reply]
The classic example of surviving a high-altitude fall is Vesna Vulović, who fell roughly 33,000 feet. However, she seems to have been inside part of an airliner at the time, which may have provided some protection/stabilisation. The highest fall, otherwise, I'm aware of is a WWII British airman, who bailed out of a bomber and then discovered his parachute didn't work. He landed in a snowy pine forest, which broke his fall enough that he survived. (On checking, Sgt. Nicholas Alkemade. There were also a couple of other allied airmen who fell from the same height and survived, on other occasions - about 20,000 feet.) Shimgray | talk | 14:08, 29 September 2005 (UTC)[reply]
The alt.suicide.holiday Methods FAQ states that one needs to fall 6 stories to have a 90% chance of dying, either instantaneously or from one's injuries. moink 23:03, 29 September 2005 (UTC)[reply]

most toxic toxin

what's the most toxic poison? ie, smallest dose can kill someone

Oxygen. No dose whatsoever is uniformly fatal to humans. — Lomn | Talk / RfC 20:45, 28 September 2005 (UTC)[reply]
According to the sci.chem FAQ (scroll down to question 10.5), tetanus toxin, botulinum toxin, and Shiga toxin lead the list, each with an LD50 around 1 ng/kg. Wikipedia's own articles give the edge to botulinum toxin and claim that the lethal dose is 300 pg/kg. Chuck 23:06, 28 September 2005 (UTC)[reply]
If you include radiation poisoning (q.v.) the fatal dose is minute in terms of the mass of radioactive material required. Shantavira 14:17, 29 September 2005 (UTC)[reply]

biggest mosque

which is the biggest mosque of the world

Shah (King) Faisal Mosque, Islamabad, Pakistan mentioned at List of mosques. the prayer hall alone is 1.19 acres. MeltBanana 21:21, 28 September 2005 (UTC)[reply]

food

what is tuna called in hawaii?

Usually "tuna". There are Hawai'ian words that are more specific: Yellowfin Tuna is "'ahi" (the first letter is an okina) and Skipjack Tuna is "aku". - Nunh-huh 04:38, 29 September 2005 (UTC)[reply]

Referencing Wikipedia

If you had to refer to wikipedia in a bibliography, how would you put it?

See Wikipedia:Citing Wikipedia. Frencheigh 23:09, 28 September 2005 (UTC)[reply]

hurricane katrina

how did katrina affect overpopulation in places like dallas and houston


   People who escaped from New Orleans fled to Dallas and Houston. Gesiwuj.

RCA/Columbia Pictures Home Video list

Hi,

I'm not sure if this is the right place to ask or even if I'm wasting your time, but I'm looking for a reference list for all the titles released on RCA/Columbia Pictures Home Video, particularly the "Magic Window" series which were issued in the custom clamshell boxes.

I've looked almost everywhere else I can think of, so I hope you can help!

Thanks, Ian Dickson Brighton UK

You're asking in the right place on Wikipedia. Columbia Pictures became part of Sony some time ago, so you could try contacting Sony in the UK and asking them. Presumably they have a catalogue with everything they've ever released on it. --Robert Merkel 13:23, 30 September 2005 (UTC)[reply]

Cities populations

I am looking for the most recent populations of the following cities, Baton Rouge,Alexandria, New Orleans,Shreveport,Lake Chareles,Lafayette,Monroe all cities are Louisiana cities.

Be careful with how you ask this one. Most recent before Katrina or most recent as of 22 September 2024 (UTC) (the date of your question)? --hydnjo talk 01:56, 29 September 2005 (UTC)[reply]
Type the name of each city into the box on the left of the screen. Press Go. That will take you to an article about the city, which will tell you a population figure (probably from a few years ago). DJ Clayworth 17:20, 29 September 2005 (UTC)[reply]

myths of authoritarianism

What are the six myths of authoritarianism?

Whatever your textbook or teacher says that they are. DES (talk) 01:45, 29 September 2005 (UTC)[reply]
Our Authoritarianism article is quite detailed but fails to state anything about 6 myths. A Google search on this phrase come up with nothing see [11] suggesting that this phrase is not widely used. I doubt if we can help you with this. Capitalistroadster 02:24, 29 September 2005 (UTC)[reply]

Grass

Is it possible to grow grass on your scalp?

If you have a good head of loam then, yes. Be sure to water and fertilize but don't shampoo. --hydnjo talk 02:00, 29 September 2005 (UTC)[reply]

how much grass did you smoke to want to ask that question? --Ballchef 03:58, 29 September 2005 (UTC)[reply]

lactulose

Does lactulose tickle?

It apparently goes great with vodka. - Nunh-huh 03:30, 29 September 2005 (UTC)[reply]

lactulose doesn't specify the tickle levels of lactulose, perhaps you should try it out? --Ballchef 03:58, 29 September 2005 (UTC)[reply]

Chocolate milk

What chocolate-flavored milk product once advertised free Captain Midnight Secret Decoder rings?

Ovaltine. Also see Secret decoder ring. --hydnjo talk 15:44, 29 September 2005 (UTC)[reply]

Swiftboating

I've noticed the use of the term "swiftboating" several times in the past few months. It is used in the sense of someone being swiftboated,ie, being lied about. Recently it was used in our local newspaper as "he was swiftboated". In that particular case the person had been the subject of a lie.

Is this word going to, or has it, entered our vocabular as a new idiom?

EWB


Fact checking

Does anyone check over newly editted pages and fact check the information added? How is one to know that the information one is receiving is true?

Please see Wikipedia:Replies to common objections and Wikipedia:FAQ. Dismas|(talk) 10:46, 29 September 2005 (UTC)[reply]

getting old

how can i get old slower?

exercise, diet, nutrition, low stress, don't smoke or drink or do drugs. amongst other things. --Ballchef 06:20, 29 September 2005 (UTC)[reply]
Considering how many things which people enjoy are bad for you, you may not live to be 100 if you abstain from all the above, but it'll probably feel like it! -- Arwel 18:48, 30 September 2005 (UTC)[reply]
Depending of your reasons... how about considering the twin paradox? :) ☢ Kieff | Talk 09:45, 29 September 2005 (UTC)[reply]

spiders2

anyone got any tips on keeping away daddy longlegs?

heavily spray the areas they like to congregate in with pesticides. Also, a daddy longlegs is technically not a spider. see Opiliones -Drdisque 06:34, 29 September 2005 (UTC)[reply]

i've been told that daddy long legs cant bite humans and only kill other spiders - so we dont kill the daddy long legs (on purpose) but let it kill the other spiders...Kattus 13:11, 29 September 2005 (UTC)[reply]

See crane fly for more information. Shantavira 15:23, 29 September 2005 (UTC)[reply]
I'd urge anyone to think once, or twice, or more than three times before getting out the pesticides for crane flies. An article on [Crane Fly Pests of the Pacific Northwest http://whatcom.wsu.edu/cranefly/] makes the point: Pesticide applications for crane flies are probably unnecessary in many cases. This misuse of pesticides is wasteful, costly and can have detrimental effects to the environment, wildlife and ourselves.. The same site provides a number of well considered treatments [12] and, FWIW, the quicklinks box at the top right seems the easiest way to navigate the site. --Tagishsimon (talk)

of course, from his question topic, he probably isn't talking about crane flies, he/she's probably talking about Opiliones (a Harvestman) or the Daddy Longlegs Spider also known as the Pholcidae for which pesticides will be effective. -Drdisque 00:35, 30 September 2005 (UTC)[reply]

laser printer vs inkjet printer

why should i get a (home) laser printer when it costs more than an inkjet and only does black and white? I need a printer that does colour and photos and is cheap, should i go inkjet? --Ballchef 06:10, 29 September 2005 (UTC)[reply]

sounds like an inkjet is best for you. Home laser printers are intended for those who print a lot of pages and/or want a printer hardwired onto their network. -Drdisque 06:32, 29 September 2005 (UTC)[reply]

thanks!

The advantages of a laser printer are that they can print much more quickly than inkjet printers, and, while their ink cartridges are more expensive, they are also larger and they last longer, so it's a better deal in the long run -- selling ink for inkjet printers is one of the biggest legal rackets of all time. They can also provide sharper grayscale prints than inkjets can. However, while color laser printers exist, they are more expensive and can't offer the quality of a color inkjet. So you kinda answered your own question; for printing photos, stick with inkjet. However, for college students, small businesses and others who need to print out lots of text -- reports, essays, research documents -- lasers can be a sweeter deal in the long run. Garrett Albright 18:16, 29 September 2005 (UTC)[reply]

Diameter of a football

That depends first on what type of football (international or American) and on the level of play (World Cup, Super Bowl, Grey Cup, kiddie league, etc). Additionally, for American/Canadian football, you'd have to specify the major vs minor axis. Anyway, the article linked above should clear this up for oyu regardless. — Lomn | Talk / RfC 13:03, 29 September 2005 (UTC)[reply]

lwas pertaining to cycle rickshwas in madras

amendments made to The Madras Hackney Carriage Act 1911 and further laws existing in the state?

Death

How exactly does a bullet to the brain or heart kill you?

to the heart, it kills you because the heart cannot attain the pressure required to pump blood to your brain because it has a giant hole in it. In the brain, it severs neuron connections required for the brain to work or the severs blood vessels that keep important parts of the brain alive. -Drdisque 16:18, 29 September 2005 (UTC)[reply]

And you question is...? --hydnjo talk 20:38, 29 September 2005 (UTC)[reply]
I think they might have been asking what the difference or what the defining point of when a sea is considered an ocean would be. Dismas|(talk) 20:53, 29 September 2005 (UTC)[reply]
Oh, sort of like when does a pond become a lake or when does a lane become a street become an avenue. There are no hard and fast rules that I could suggest to the questioner so I bounced the question to get a more specific question. I suspect that Dismas is correct as to the root of the question, I was just hoping to draw it out in a more specific context. --hydnjo talk 23:10, 30 September 2005 (UTC)[reply]
I've always heard that seas are described by the land that surrounds them, while land is described by the oceans that surround it. -Tim Rhymeless (Er...let's shimmy) 00:10, 2 October 2005 (UTC)[reply]

Does This Energy Drink really work?

I was wondering if XS Citrus Blast Energy drink is simply made up of caffeine and some flavors, or does this drink have a makeup of things that really provide energy. Also, would it be best to just buy a can of coke instead?

It depends on how you define energy. By most definitions any drink with any amount of calories can be described as an energy drink. I assume this drink has plenty of sugar. Whether you would be better off with a can of coke depends on the relative costs and your taste preference. A handful of sugar in a glass of water would be even cheaper. alteripse 16:25, 29 September 2005 (UTC)[reply]

smallest state in the USA

What is the smallest state in the USA?

That would be the State of Rhode Island and Providence Plantations. — Lomn | Talk / RfC 16:02, 29 September 2005 (UTC)[reply]
Rhode Island is the smallest in area. Wyoming is the smallest in population. Chuck 16:01, 29 September 2005 (UTC)[reply]

Lional train

What is the value of a Lional engine and coal tender, model 736?

try searching for recently sold examples of such an item on ebay -Drdisque 16:15, 29 September 2005 (UTC)[reply]

baseball

why did they stop selling alochol after the 7th inning.And what year and teams were playing

They stop selling alcohol then to prevent fans from getting as drunk and to give drunk fans time to sober up before they leave (assuming they leave at the end of the game). The second part of your question doesn't make any sense. -Drdisque 18:00, 29 September 2005 (UTC)[reply]

I think the original poster meant that they wanted to know what year they stopped selling beer after the 7th and what teams were in MLB at the time of the new rule. Although I don't know the answer to that, I think that's what they meant to ask. Dismas|(talk) 18:55, 29 September 2005 (UTC)[reply]

help me please !!!

i have just started a course at college but i need help can somebody tell me what the word demonstrator means in relation to the famous playwright bertolt brecht ??!! thank you lisa-kate 212.126.144.12--212.126.144.12 18:29, 29 September 2005 (UTC)[reply]

Have you tried reading our article on Bertolt Brecht? --Sherool 18:57, 29 September 2005 (UTC)[reply]

Demonstrator can have several meanings depending on context. One meaning is "a person who participates in a public protest," especially someone who carries a sign to raise public understanding of the issue. Brecht was more of a political activist than most playwrights. This is a guess. alteripse 10:40, 30 September 2005 (UTC)[reply]

VCR

How long do you think it'll be before they stop making VCR's?

Decades, if ever. You can still buy brand new turntables, and digital video recorders are not nearly prevalent enough yet to phase out VCRs. — Lomn | Talk / RfC 20:01, 29 September 2005 (UTC)[reply]
The major electronics store I went to about six months ago no longer sold stand alone VCRs, only combination DVD/VCRs. Cheap enough though, under $100. --hydnjo talk 20:31, 29 September 2005 (UTC)[reply]
To summarise, VCRs will be around in small quantities in specialised applications for many years (if for no other purpose than to enable access to historical recordings). However, they will disappear as a mainstream consumer item before too long; the process already has begun. --Robert Merkel 13:18, 30 September 2005 (UTC)[reply]
Videoape is still used heavily professionally — before the day when VCRs are no longer available we will reach the day when VHS is gone and Betacam is still in use! — File:Ontario trillium sig.pngmendel 19:14, 30 September 2005 (UTC)[reply]
We haven't really answered the question. I think that a small volume new VCRs will be manufactured at least through the next 8-10 years. I don't think any will still be produced 15 years from now. ike9898 00:01, 2 October 2005 (UTC)[reply]

Baseball Rules

Is there a rule in baseball that states, that if the game is tied, with bases loaded and it's the bottom of the 9th inning, you can't intentionally get hit with the ball to drive in the winning walk?

In baseball, a batter may not intentionally step into a pitch, or fail to attempt to avoid a pitch (to draw a hit-by-pitch), at any time; he is out if he does so. However, a regular hit-by-pitch may occur at any point. See the official rules, section 6.08[13]Lomn | Talk / RfC 19:58, 29 September 2005 (UTC)[reply]

Iraqi Dinar

On the contents page under 2.4 ,value of the new dinar, there is an urgent update notice that says the Iraqi Dinar is expected to oeg the week of October 1st 2005. Can you explain why?

Mallary

The notice is badly written, granted, but it basically says that the dinar is expected to be pegged to the US dollar at the beginning of October. This means that its exchange rate with the dollar will not be allowed to fluctuate. --Gareth Hughes 20:37, 29 September 2005 (UTC)[reply]

shelters ANDERSON

i have been reseraching material for a book on me during the Blitz of Plymouth. You are incorrect on the number of people for a shelter. Each sheet of metal was multiplied accourding to who were to use it. One adult & 2 children equaled ONLY 2 sheets deep! Two adults or more, 3 sheets.

Father away in the Navy, so the two sheet deep size was our lot! Floor was wood made by grandfather to keep out the damp. A FULL SIZE DOUBLE MATRESS FITTED PERFECTLY ON THE FLOOR. My father cut a naval hammock into two & hooked to the length of the 2 sheets. We swayed as the gun vibrations from antaircraft weapons fired from local park.

I was later to be a Sgt. in the WRAF in 1950's. Meeting many vets from the war era.

Thanks for your suggestion. When you believe an article needs improvement, please feel free to change it. You can edit almost any article on Wikipedia by just following the Edit link at the top of the page. We encourage you to be bold in updating pages, because wikis like ours develop faster when everybody edits. Don't worry too much about making honest mistakes—they're likely to be found and corrected quickly. You can always preview your edits before you publish them or test them out in the sandbox. If you need additional help, check out our getting started page or ask the friendly folks at the Teahouse. Dismas|(talk) 22:27, 29 September 2005 (UTC)[reply]

Good Burger

In what fictional Florida town is "Good Burger" filmed?

The movie can't be "filmed" in a fictional town. According imdb.com, the movie was filmed in West Covina, California. Couldn't figure the name they gave to the fictional town. ☢ Kieff | Talk 01:19, 30 September 2005 (UTC)[reply]

focus strategy

baseball hall of fame

How many hall of famers are pitchers,catchers,infielders,outfielders,managers,and umpires?

Mt. Wikipedia

When website is loading, background has the outline of a mountain, I'm assuming a mountain of knowledge? Ararat? Sinai? Paramount Pictures? Basically--What Mountain?

  • Actually, if you look more carefully, it's a picture of an open book. But I do agree, "Mt. Wikipedia" has a nice ring to it. Are there any suitable elevatated spots around Tampa Bay?--Pharos 23:57, 29 September 2005 (UTC)[reply]

The highest elevation in Florida is a large Landfill near Miami. -Drdisque 00:28, 30 September 2005 (UTC)[reply]

I actually first read this as "Mr. Wikipedia". JIP | Talk 10:50, 30 September 2005 (UTC)[reply]

Unicorn Story

Years ago I read a story about Noah's Ark and the Unicorn. In a nutshell, it was about Noah rounding up all the animals and seeing the unicorn in the distance, and every day that went by, he'd round up more animals but the unicorn would never come close. In the end, the "legend" says that the reason why unicorns are no longer around is because he chose to stay in that world than face this one. Noah was never really able to convince the unicorn to come aboard. It's such a cute story, but I can't remember either the title or the author. Has anyone read it? Does anyone know who the author is or what the story is titled? Vanessa

Also, there's a song called "The Unicorn" by The Irish Rovers with lyrics by Shel Silverstein that tells a similar story. moink 19:18, 1 October 2005 (UTC)[reply]

Who?

Hi. Im Hoosay. I have a question about your "Hurricane Rita" website.I'm writing a five paragraph essay about natural disasters in core class and we have to write where we got our information from,the articals title,the webpage,internet address and the author.I could'nt find the Author(s).Would you please send it to me?I don't have an email address so will you please send it to me by mail!

You can get help with this at Wikipedia:Citing Wikipedia. Joyous (talk) 00:47, 30 September 2005 (UTC)[reply]

looks

y r people funny looking

Because we can't all be dolphins. — Lomn | Talk / RfC 04:18, 30 September 2005 (UTC)[reply]

Refineries

When was the most recent oil refinery built in the United States and where?

according to this article, the last refinery in the US was built in 1976, in Garyville, Louisiana. Gentgeen 03:07, 30 September 2005 (UTC)[reply]

http://explanation-guide.info Largest Mobile Companies BIASED!

Why is explanation-guide.info's of the Largest Mobile companies web page biased? It states at the bottom of the page that your web site gives them the info, but all of the US carriers stats are a year and more out of date. They list all of the EU and Asian companies as being the largest, even thought Cingular, Verizon Wireless, and Sprint-Nextel have larger numbers then most of the other EU and Asian related carriers on their web site. And why will they NEVER replay back to my question about this? Can you please help me out?

Cingular 51 Million users Verizon Wireless 47 Million users, and Sprint-Nextel 45 Million Users as of the last quarter in 2005.

How can these other carriers have higher numbers?! Why can't you and your partner web site give credit to the USA?!

Orange (GSM) - 40 million mmO2 (GSM) TIM (GSM)

Thanks, Russell

  • Firstly, all the information on Wikipedia is licensed under the GFDL, and may be used by anyone freely as long as the source is credited. As such, explanation-guide.info isn't a "partner" website of ours - we have nothing to do with them (just to make that clear!), and they simply take our information and republish it.
  • Secondly, as far as I can tell they're using an old copy of our database, which hasn't been updated since... hmm... examining the history of some articles, they seem to have a copy from around last August or September - so at least a year out of date. If you look at our List of mobile network operators, you'll see it's pretty up to date, and doesn't appear to have any anti-US bias (Cingular is still only the eighth largest in the world; I suspect this is because foreign operators break into the US market easier than US ones break into foreign markets, but couldn't really say).
  • Thirdly, the reason they're not replying to your emails... hmm. Many of these sites are basically just there to make some ad money - you'll note they have google ads through their site - from people searching for information, and aren't particularly concerned about anything else. Not to say that this particular one is - I've never heard of them before - but, well, it's a possibility. Shimgray | talk | 02:26, 30 September 2005 (UTC)[reply]

Micro Economto the marketking ics

Why are consumers the

Well the best answer I can give you is

who was the first 2 million pound player in the uk football leagues

I don't know, but it's made me start an article called British football transfer record. I can't find it on the net, anywhere. Trevor Francis was the first million pound player, in 1979. Man Utd's purchase of Bryan Robson (£1.5m) in 1981 took the record. But the next 'record signing' I could find was Roy Keane (£3.5m) in 1993, and there must have been a £2m player between 1981 and 1993. If anyone knows about football transfer fees, help out with the article I started, for the love of God. Proto t c 11:18, 30 September 2005 (UTC)[reply]

What is mineral wall fire insulation?

Health Psychology

Define & discuss the mind-body split.


155.232.250.51 11:21, 30 September 2005 (UTC)[reply]

Why not read our article on psychology and its associated articles, read your text book, and then get back to us with a more specific question regarding your homework? Dismas|(talk) 11:27, 30 September 2005 (UTC)[reply]

Palooza!!!!

I have got in trouble on this site for teying to get the answer to this question so i really hope someone can help me!!! What DOES the word palooza mean? I have heard it several times recently and it made me curious. If you have any idea please let me know. On my talk page or whatever, thanks. Skooky 13:21, 30 September 2005 (UTC)[reply]

I'm not sure, but I'd guess it's derived from Lollapalooza. Although Lollapalooza became well-known as a music festival which started in 1991, the word itself has been around since at least 1904, and originally meant "remarkable or wonderful person or thing." Since Lollapalooza (the music festival) started, I've occasionally heard "-palooza" applied as a suffix to something else to indicate some kind of festival, but I haven't heard "palooza" used as a word on its own. Chuck 20:44, 30 September 2005 (UTC)[reply]

The Letter People --

I am trying to find tapes/CD's/DVD's of "The Letter People" series that aired on Public Broadcasting in the 70's. It's my understanding that New Dimensions in Education, Inc. sold the rights in 1990. I am not interested in the badly copied versions that are being sold on Ebay. SR


Russian Companies in US

Where do I find listing of Russian companies doing business in the US? Thanks MB

List of Fictional Florida Cities, Villages and Towns

Can you tell me the list of fictional Florida cities, villages and towns?

Well, there's Andaerixodortilka, Florida, which is a fictional Florida town, at least in a two-sentence story I just made up which currently exists only in my mind. Seriously, I don't think such a list exists because of the difficulty in compiling it, unless you want to narrow the scope of your question--fictional Florida towns in a particular book? movie? TV show? something else? Chuck 17:28, 30 September 2005 (UTC)[reply]

Univision

Was Univision always in Spanish?

Sí. Mira Univisión por mas datos. Garrett Albright 17:04, 30 September 2005 (UTC)[reply]

Date: February 10, 1917

I did a Google Search on a lady named Myrta Belle Strong. One of the results I got back was from WIKIPEDIA and listed that on February 10, 1917 she married Carl L. Gregory. I am researching the life and career of Mr. Gregory and while I have seen News reports of His marriage to Ms. Strong, I have never been able to get an exact date, nor have I been able to confirm the marriage. I am looking for the source of your information ot verify the date of this marriage.

Thank you.

Buckey Grimm

Looks like you've found one of the weaknesses of Wikipedia. Although authors are encouraged to cite their sources, so that people with questions like yours can verify the information in the article if they want to, authors are frequently lazy and do not cite their sources. In fact, that is the very first objection listed on Why Wikipedia is not so great. You might have better luck posting your question to the Talk Page for the article where you found the information (I'm assuming it's 1917; that's the only place I was able to find mention of that marriage) where the person who added the information is more likely to see your question. Chuck 17:24, 30 September 2005 (UTC)[reply]
Both Carl L. Gregory and Myrta Strong attended Geneva High School in Geneva, Ohio (he in the Class of 1900, she in the Class of 1902). [17]. He is the correct age to be the same person as Carl L. Gregory, the cinematographer (1882-1951) born in Kansas, died in California, who is listed on IMDB as having married Myrta Belle Strong on 10 February 1917, and as having married Marie Garrison afterwards. [18]. (Meanwhile, if we don't have an article on either Carl or Myrta, why are we listing their marriage on our dates page?). - Nunh-huh 02:19, 1 October 2005 (UTC)[reply]

Good Burger

What is Good Burger's fictional address?

Parts of a ship

Why is the control center of a ship called The Bridge?

See Bridge (ship). — File:Ontario trillium sig.pngmendel 18:58, 30 September 2005 (UTC)[reply]

Girlfriend

How do I get a Girlfirend, I am a boy of 15 and based in the UK?? nmak3000

Bathe every day, wear nice shoes, tell them they look nice even when they don't, and get a job. -- Finlay McWalter | Talk 19:01, 30 September 2005 (UTC)[reply]
Good advice. Also, don't get discouraged if your efforts are not reciprocated the first time. Be optimistic and be friendly in lots of directions. When some one is interested back, you will probably have no problem recognizing it. alteripse 19:48, 30 September 2005 (UTC)[reply]
Confidence is pretty much key. Most women (and girls), even if they wont admit it, find confidence attractive. If you don't feel confident, act it anyway, and eventually you will become it. But never be cocky or arrogant. So basically what I am saying is ask, and be willing to be ok with them not being interested. Start small, ask them to a movie or something, don't ask them to be your gf right off the bat. Beyond that, have something interesting to talk about. Mostly only talk about yourself if she asks, otherwise try to learn and talk about things she would be interested in. You'll make all kinds of mistakes no matter what. You might as well get them out of the way sooner rather than later. - Taxman Talk 13:08, 1 October 2005 (UTC)[reply]
Tell a girl she's beautiful. Teenage girls all are, they don't hear it enough, and they mostly have low self-esteem. moink 19:13, 1 October 2005 (UTC)[reply]
Find a girl you share an interest (music, films, books, quantum mechanics) with, a subject you can talk about with them at length without either of you getting bored. The rest will hopefully fall into place. --Sum0 20:29, 1 October 2005 (UTC)[reply]
Don't be the guy who is 'obsessed' with or 'worships' a particular girl he has no real relationship with. You might think obsession is romantic, but usually it just scares the girl (and her friends) away. ike9898 23:51, 1 October 2005 (UTC)[reply]

Les Rythmes Digitales

Does anyone know the lyrics (assuming it is sensical) in French? (that is if it is sensical)?

Programming in Fortran

Hello there, Please I will like to get answer for the following questions. I was ask to write programm that solve the question. I was given 30 question which I have been able to answer some but these are giving me probelm. Please will appreciate if i can get answers to the question:

1- write a valid program segment to demonstrate the usage of the following: i- List-directed input-output statements ii- Data statement iii- Equivalence statement iv-named common v- blank common vi- block common

2- company xyz gives no discount on a bill(in dollar) of less than 20,000, 5% discount on a billfrom 20,000 up to a value less than 50,000, 7% discount on bills from 50,000 up to a value less than 100,000, and 10% discount on a bill of 100,000 or more. write a program that can be used by the company to compute and print the discounts on the customers bills.

3- A trader has five store in a state. the store makes sales 7 days a week. the daily sales are in an array called SALES. Write a program i to read the whole array from keyboard into the computer memory ii compute and prints the total slaes for each day iii compute and print the total weekly sales for each store iv compute and print the total weekly sales for all the store.

4 write a program that compute the sum of all prime members between 1 to 1000.

5 write a program that compute the overall resistance of four resistor connected in parallel.65.172.4.250 19:26, 30 September 2005 (UTC)[reply]

We don't answer homework questions outright here, but we may be able to give you a few tips. I'm not much of a programmer myself, so I won't try to write specific code for these, but for number 4 you will need a method to find the prime numbers between 1 and 1000 first; one such method is the Sieve of Eratosthenes. For number 5, you will need to know the formula for resistors in parallel, which can be found at resistor#Series and parallel circuits. Chuck 20:12, 30 September 2005 (UTC)[reply]
I am a programmer but it has been decades since I programmed in FORTRAN. In general two important ingredients:
  • How would I solve some problem manually, on paper without the computer? This is also important when testing the reasonableness of the answer from the computer program.
  • Are there some features of the programming language, such as recursive that make it easier to solve with this or that programming language than doing it human manually?

AlMac|(talk) 04:58, 3 October 2005 (UTC)[reply]

Japanese ppl

Why are Japanese ppl so tiny?

Oh, do you mean like these Sumo wrestlers? --hydnjo talk 20:45, 30 September 2005 (UTC)[reply]
God you got served.

Wing Clipping

Two weeks ago I bought myself a pet cockatiel... I love my pet but I'm trying to train him and I've read repeatedly in several webpages that the first thing one must do is clip a bird's wings. Although I find this a cruel thing to do, I am thinking of taking him to a vet and having his wings clipped. How often should this be done though? How long before they "grow back"? Also, is it something I can do myself? I think it'd be a good idea to take him to a vet first to have it done professionally, but can I learn to do it myself or is it too risky?

wing clipping, if done correctly doesnt hurt the bird - it is rather like cutting one's own fingernails... you have to make sure that you know where the bird's bone and flesh ends, so as you do not hurt the bird... you can do it yourself, but if you are unsure, then yes go to the vet and they'll probably show you how to do it... in terms of how long before trimming again, well you can tell yourself usually, the birds wings look normal instead of cut straight - and they can fly again....we've been clipping our birds wings for years...hope this helps you ... Kattus 00:46, 1 October 2005 (UTC)[reply]
To make it painfully clear: the only thing that is cut when wings are clipped is feathers. And the only part that is clipped is already dead. By all means let a vet show you how to do it rather than try it by yourself the first time. see here - Nunh-huh 02:06, 1 October 2005 (UTC)[reply]
See also our article on wing clipping. Samw 12:30, 2 October 2005 (UTC)[reply]

Searching "POPO" the Clown

Searching for information about "Popo, the Clown". What circus("}, what years, etc. Signed, Gregory D. Harper

Two tier stock take overs.

An English royal woman...Need a name!

The information I have is this: She married Thomas, she fell in love with a footman/soldier, Thomas learned this and killed the footman, buried him outside the castle wihout telling her, she died and haunted the castle (grey lady? Louisa?), Thomas moved away to escape her ghost, was then buried in the same cemetary as his victim.

I think this is somewhere in Westminster or Glastonbury.

Thank you in advance for your help.


Thomas (Thynne), 2nd Viscount Weymouth, and his wife Louisa née Carteret. We find the story, a bit mangled, here: "In Southwestern England the 16th century Longleat house stands. A ghost story is told here. Thomas the Second of Viscount, married Louisa Carteret. He was as evil as she was sweet. He suspected her of having an affair with a footman and murdered him with the help of two servants, and hid the body. Louisa died not long after, they claimed, of a broken heart. Thomas started to see her ghost, smell her perfume, and watch as things would move on their own-he fled in fear and never returned. People thought it was just a legend even though her ghost was seen countless times by many. Three centuries later, workmen dug up some flag stones and found a corpse dressed in 18th century attire. The legend was proven to be true. They buried him in a graveyard not far from the house, and from the tomb of Thomas. Murderer and victim together for all eternity. "The Gray Lady of Louisa" as she is now called, still haunts the house to this very day."

Of course, there's no "Second of Viscount", but the husband of Louisa Carteret was 2nd Viscount Weymouth. With this, we can find the tale retold better here, and with less bodice-ripping:

"Longleat, the residence of the Marquess of Bath, has been haunted for many many years. It is said that his ancestor, Thomas Thynne, 2nd Viscount Weymouth, had fought a duel in a passage at the top of the house with an unknown man, who was rumoured to be his wife's lover. He killed this man, and had him buried in the cellars of Longleat. The passage where the duel is said to have been fought is now known as 'The Green Lady's walk', and is said to have been haunted by the spirit of Lady Louisa Carteret, wife of this 2nd Viscount Weymouth. When central heating was put into Longleat, during the 5th Marquess's lifetime, the body of a man was found buried in the cellars. He was wearing jackboots, which crumbled away as soon as the body was exposed to the air. Lady Louisa Carteret, Viscountess Weymouth, died aged about twenty-two, after three-and-a-half years of marriage and nine days after the birth of a child. It was rumoured that Viscount Weymouth nearly ruined himself and his two sisters by his extravagance after the death of his wife."

- Nunh-huh 02:00, 1 October 2005 (UTC)[reply]
You probably misread Westminster for Warminster. --Gareth Hughes 17:03, 1 October 2005 (UTC)[reply]

Thanks for all the info. I really need to find a picture of her (or portrait). I sa it briefly on TV, but I need it again! Thanks for the help again!

How to ask a Judge to make a jugement on a matter.

1st. October 2005.

can i ask a high court judge to make a legal judgement of a concern of mine, as to the conduct of a Local Council?

i hope you are well, your servant, Br. Michael.

The Hermitage 20, Berwick Road Marlow Buckinghamshire. SL7 3AT. Grande-Bretagne.

Tel - Fax +44 01628 474034. from sunrise to sunset.

Hermits Journal. www.marlowhermitage.net

To get a judge to do it, you need to apply for Judicial Review. The standard for that is very high (meaning a judge is very loathe indeed to overturn anything that a democratically elected council has done). You're rather more likely to get some headway from the local government ombudsman. Even then, democratically elected councils enjoy considerable leeway, so you'll need to show they did something egregiously wrong before you have any chance of changing things. -- Finlay McWalter | Talk 21:57, 1 October 2005 (UTC)[reply]

Football Double Header

In 1948 or 1949, when I was a young lad, my Dad took me to see the one and only double header football game in Cleveland Ohio. This was a pre-season exhibition trial double header, never to be repeated. The Cleveland Browns were involved, but I can't remember who the other 3 teams or the scores were. I would appreciate it if you could let me know. I am 74 yrs old, and disabled. I have tried in the past to get this info without any success. This would also make a good trivia question for Jeapordy. TNX Norb Walker Sat Oct 1, 2005 <bronreklaw a t hotmail d o t com>

Google is very good at these sort of things. Searching for "cleveland football double header", the third link is to http://answers.google.com/answers/threadview?id=567111. According to Google Answers, it was 1963, with the Giants beating the Lions 24-21 in the first game, followed by the Browns losing to the Colts 21-7. -- Rick Block (talk) 15:07, 1 October 2005 (UTC)[reply]
That doesn't seem quite right..that's an almost 15-year difference in the timeframe...could that be mistaken somehow? Tim Rhymeless (Er...let's shimmy) 23:57, 1 October 2005 (UTC)[reply]
According to the Cleveland Browns official site, the team only played one preseason game in Cleveland in the 1940s, a 21-21 tie with the 49ers. Every other preseason game they played in the 1940s was either at Akron or Toledo. This page doesn't indicate whether the 49ers game was part of a doubleheader. --Metropolitan90 04:40, 2 October 2005 (UTC)[reply]
I'm afraid to tell you that preseason doubleheaders continued in Cleveland into the 1960s. One game would feature the Browns and another game two other teams. According to this guy, the games were played from 1962-61. In the book The Cleveland Browns: The Official Illustrated History, you can see a picture of a game at the old Stadium in which the Browns weren't playing. It looks like Atlanta vs. Minnesota. Mwalcoff 20:04, 2 October 2005 (UTC)[reply]

Acronym in different scripts

I would like to write the acronym KIS in different scripts, especially Korean, Japanese, Hebrew, Russian, Greek, Hindi and Tamil. Does anyone know where I can look up this info. Thanks.

There are some links on Transliteration to script-specific pages. Ojw 16:53, 1 October 2005 (UTC)[reply]

Hello there, I was hoping that you might be able to add my shop to your links. I would be able to advertise your site with every purchase made from my shop, and also on the flyer i have produced to deliver to all known petrol garage's in the UK (of which i have nearly compiled a list). The link to my shop is: http://stores.ebay.co.uk/Make-your-car-a-supercar-with-Nitro, and although i will not be able to put a link for your site in my shop, the coverage that i will produce via my flyer will be constant and very wide-scale. I eagerly await your response to this email request. Many thanks, Marc Chilton.

PS. My email address should you wish to respond is: (email removed)

Wikipedia is not the place for advertising. Thanks for your suggestion though. Dismas|(talk) 14:38, 1 October 2005 (UTC)[reply]

Making Soap Operas On Microsoft Word

How many pages are required in order to make a full soap opera on Microsoft Word?

Write 5000 words. Use the "word count" feature of word to know when you arrive. lots of issues | leave me a message 16:53, 1 October 2005 (UTC)[reply]

which is the fastest car in the world

The fastest wheeled land vehicle is the ThrustSSC, the fastest street legal car is generally regarded to be the McLaren F1 -Drdisque 18:45, 1 October 2005 (UTC)[reply]

We have a new "World's fastest production car"; the Bugatti Veyron, which has just been released and was timed at 407 km/h. A complete waste of time of a vehicle (on most racetracks I'd back a 911 GT3 against it, let alone the McLaren), but it is mighty fast in a straight line if you're suicidal enough to try. --Robert Merkel 14:38, 2 October 2005 (UTC)[reply]

History

can I see a picture of Aksapada Gautama

In general, Google Image search at http://www.google.com/imghp is good for these sorts of things, but a search for "Aksapada Gautama" doesn't come up with anything. Are there any alternate spellings? User:Zoe|(talk) 21:35, 1 October 2005 (UTC)[reply]

Half-Life multiplayer statistics

I seem to recall a website somewhere out there that gathered statistics from a great number of Counter-Strike/Day of Defeat (possibly other Half-Life mods) servers and collected them into a database, and just by searching for your multiplayer user name (e.g. "Sum0") you'd get a list of your total number of kills, deaths, favourite servers, and so on. But for the life of me I can't find it again. The URL possibly had "athletics" or "athlete" in it.

Thanks, Sum0 20:20, 1 October 2005 (UTC)[reply]

Ask-a-question web sites

I know you can ask questions here on Wikipedia but would like to know if there any web sites that let you ask questions and then get answered. I tried searching on Google but it came up with FAQs for different issues. Any help? -- Thorpe talk 20:36, 1 October 2005 (UTC) [reply]

You could try asking on the Straight Dope forums (or ask Cecil directly). --Sum0 20:53, 1 October 2005 (UTC)[reply]
"Goolge Answers" is a service where you pay to have your question answered. I think that Yahoo has a free question answering thingy. ike9898 23:39, 1 October 2005 (UTC)[reply]
Experts-exchange is a good site for asking IT-related questions. I haven't been there in a long time but it used to be a decent resource. Rhobite 23:51, 1 October 2005 (UTC)[reply]
If you don't mind what seems to me like a spectacularly bad signal to noise ratio, you might have a look at Wondir. - RedWordSmith 07:09, 2 October 2005 (UTC)[reply]
It's not the Web, but there's always Usenet and mailing lists. — File:Ontario trillium sig.pngmendel 21:32, 2 October 2005 (UTC)[reply]

What do chickens eat?

It's impossible to resist answering "chickenfeed". However, we will generously point out that chickenfeed was traditionally grain (like bird seed). We suspect that some sort of industrial byproducts whose ingredients we really don't want to know have replaced chickenfeed in large poultry factory farms. alteripse 22:48, 1 October 2005 (UTC)[reply]

In modern agriculture chicken feed may be made from many ingredients, usually formulated to provide the chicken with the nutrients it needs for rapid growth, a the lowest price. I'm no expert, but I think some common ingredents would be corn grits, soybean flakes, and meat & bone meal. ike9898 23:36, 1 October 2005 (UTC)[reply]
According to the tags from the bags of chicken feed that we recently bought, Grain products, processed grain by-products, plant protein products, calcium carbonate, monocalcium phosphate, salt DL-Methionine, choline chloride, and about 20-30 more things that I can't really pronounce. If you want to know more you could find a feed store in your area maybe. Also the companies that sell chicken feed probably have the ingredients on their web sites. Dismas|(talk) 11:59, 2 October 2005 (UTC)[reply]

Best season to start a new lawn from seed

What is the best time of year to start a new lawn from seed? (I live outside of Philadelphia, PA). Would it die in the winter if I planted the seeds in the fall? ike9898 23:32, 1 October 2005 (UTC)[reply]

I planted a big-ish patch in April, and now it's indistinguishable from the grass surrounding it (after several months of looking like a bad hair transplant). If your ground is at all impacted, it's probably a good idea to give it a good forking first. -- Finlay McWalter | Talk 23:47, 1 October 2005 (UTC)[reply]
Fall is an excellent time to start a lawn (in the Northeast) because:
  1. No weed seeds are trying to establish during the fall season,
  2. The ground is warmer than springtime and thus encourages faster germination of the grass seed,
  3. Cooler air temperature results in less evaporation (easier to irrigate),
  4. Springtime "mud" season is more likely to do damage to seedlings just by walking around.
  5. Seed planted by early October will have survivability through winter if it has been "winter "fertilized,
  6. The Winter/Spring frost heaving will loosen the soil and promote growth of existing seedlings in springtime,
Having done both Spring and Fall lawn starts, I would definitely recommend a start in early September as optimum. Put down 50% more seed than recommended along with starter fertilizer and granular lime. Good luck is also an important ingredient along with plenty of water for the first few weeks. --hydnjo talk 03:57, 2 October 2005 (UTC)[reply]

gis attribute

What exactly would you like to know about GIS attributes? Which software package are you using? What feature(s) are the attributes relating to? etc. Please be as specific as possible in order to help us help you. Thryduulf 21:51, 2 October 2005 (UTC)[reply]

Miss Vero's Supermela

Anyone know to what this refers? May come from the period 1915 to 1945. Term/phrase appears in recollections, written in 1972, from an aunt born in 1899, deceased in 1992.

Generally, the reference desk struggles a bit with this sort of question. If you can supply some other context - for instance, where your aunt lived, and in what context the reference appears, it may help. --Robert Merkel 14:17, 2 October 2005 (UTC)[reply]

N.Y. Times

Who owns the n.y. Times

See [19]--inks 00:47, 2 October 2005 (UTC)[reply]

births in the 1950s

(preceding unsigned comment by 68.113.141.22 (talk · contribs) 02:03, 2 October 2005)

If you want a general list of people born in the 1950s, see Category:1950s births. If you're looking for information on general demographic trends, you might read Post-World War II baby boom. It would help if you were to phrase your question as a question so the people who answer it can figure out what exactly you want to know. —Charles P. (Mirv) 02:22, 2 October 2005 (UTC)[reply]

You could also try 1950#Births through 1959#Births --Borbrav 02:28, 2 October 2005 (UTC)[reply]

Drowning

How exactly does drowning kill you? Why can't the body take the oxygen from the water?

If you try to breathe water the water will fill your lungs causing them to explode in a gorey mess. If they could take out the oxygen, what will the lungs do with the hydrogen? Make a bomb? Not likely.
I think a more rational answer is that the concentration of (dissolved) oxygen in the water is too low for our lungs to process. I remember reading years ago that fish die out of water due to a similar problem -- it's not that they can't breathe the air but rather that their gills collapse, leaving them with insufficient tissue surface area to get enough oxygen. Regardless, bombs and hydrogen have nothing to do with the answer. — Lomn | Talk / RfC 06:39, 2 October 2005 (UTC)[reply]
There's an article on Liquid-breathing which discusses liquids with dissolved oxygen. Perfluorocarbon is used, rather than water. Ojw 13:46, 2 October 2005 (UTC)[reply]
If you're meaning taking out the oxygen from the water in terms of 2H2O->2H2+O2, then the problem is in that it would require large amounts of energy for the organism to perform this reaction, more than what is derived from oxidising food. --Borbrav 22:55, 2 October 2005 (UTC)[reply]

Chicago Cubs/Chicago White Sox Question

Which mediocre player did the Sox trade to the Cubs who then went on to be a huge success?

Body swapping

Is it actually possible to swap bodies with someone? By like brain transplant or some magical way? And would anyone be interested in writing a list of all the places this cliched plot point has happened?

We have an article on Whole-body transplants that may be of interest. While there are major technological hurdles to overcome and such an operation is not possible today, it may be possible in the future. One major hurdle to overcome is connecting the brain to the new body (consider the difficulties experience by those with a severed spine - almost always permanent paralysis). Robert J. White once connected the brain of a dog to the body and circulatory system of another dog (see the first external link in that article, and also note what it says about the monkey head transplants). Hope that helps. - RedWordSmith 06:56, 2 October 2005 (UTC)[reply]

website of satirical newspaper

This is bugging me soo much!! There's a website for a newspaper (I think it's a real life one) that's basically a satire of old-fashioned turn of the century papers. It had ridiculous patents (for some reason the only one I can remember is a Dead Crow in a Box? or something) and advice, I think? And strange rambling editorial columns from a sailor. It has a really generic name, like the Weekly or something. I lost the URL before, and found it, and now I've lost it again. D'oh!

Please help me, I'm soo frazzled from constant googling :) Thanks!

I'm not completely sure this is the right answer to your question, but you might like the historical issues of The Onion. David Sneek 07:53, 2 October 2005 (UTC)[reply]
The obvious guess would be Private Eye, but try some of the listings at Satire and Category:Satirical magazines. Ojw 13:39, 2 October 2005 (UTC)[reply]

Hi,

Not a complaint, just a warning - the page about Serb enclaves in Kosovo links to a page about Gracanica in Bosnia (rather than Gracanica in Kosovo).

Cheers.

Keep up the good work,

Sam

Thanks, I changed it. David Sneek 09:54, 2 October 2005 (UTC)[reply]

Time changes in Nigeria

Hi,

I'm trying to find out if there has ever been a time change in Nigeria over the last 50years. By time change I mean a change in the local clock setting (not related to day light savings....which nigeria does not have anyway. Would anyone be able to help me please?

Thanks

Dils

According to Nigeria's entry in the "africa" file which is part of the timezone data incorporated into most unix-like operating systems (the most recent version is available from ftp://elsie.nci.nih.gov/pub/tzdata2005m.tar.gz), there have been no changes since September 1919 (when Lagos switched from local mean time to the present UTC+01:00 time zone). —AlanBarrett 16:14, 2 October 2005 (UTC)[reply]

How do you get rid of a Bees Hive in your home?

We have a small gap that went un noticed and a colony of bees made a nest. Should i wait for the cold weather to set in before I attempt to exterminate? I was told that they die the next season.68.170.197.142 12:52, 2 October 2005 (UTC)[reply]

It'd probably be safest to hire an exterminator. — ceejayoz .com 13:52, 2 October 2005 (UTC)[reply]

ranking in the british police

ranking in the british police

indian defence

If you're meaning India's national defence system, try Military of India --Borbrav 22:40, 2 October 2005 (UTC)[reply]

operations by indian defence

Describe and evaluate three models of abnormality

Please be more specific. Do you mean the Queen's Indian Defence, the King's Indian Defence, the Nimzo-Indian Defence or perhaps the Bogo-Indian Defence? David Sneek 16:06, 2 October 2005 (UTC)[reply]

BBC TV Show

Does anyone now what the latin phrase at the beginning of the 90's childrens show Aquila was? It was repeated in the intro every episode --82.44.216.80 15:29, 2 October 2005 (UTC)[reply]

Getting rid of Insects

How do you get rid of Box Elder Bugs?


Components of Business Environment

What are the components of business environment?

Environmental Analysis

What is the concept and purpose of environmental analysis? What are the techniques of environmental analysis?

Buffalo, N.Y. - snow/weather?

Does one part of Buffalo, N.Y. get more snow than another area? My daughter is moving to the area and I would like to know which side has the least snow fall. Thanks.

Kat

Amateur musican performing for tips.

I am considering setting out on the street corners and byways where I live and playing my guitar and singing songs that I've learned from various mediums and "passing the hat" for tips. Is this legal? I'm not really destitute or anything by any stretch of the imagination nor am I in a needy financial position. But just wondering if anyone out there could offer some insight that could possibly save me some greif later on?

Many cities require a street performer's licence. Also the cops may complain if they feel a crowd who have assembled to watch you are causing an obstruction. Some places have formal or informal arrangements for the distribution of busking slots; it's not impossible that someone else will show up and complain that you're in what he feels is his spot. You should probably check the bylaws of the city in which you plan to play, and maybe ask in a local music shop (who may know the ropes). -- Finlay McWalter | Talk 18:13, 2 October 2005 (UTC)[reply]

Public enterprise in India

What is public enterprise? Discuss the role played by public enterprise for the economic development of India.

Why not start out by reading the articles on public corporation and India and then research your homework questions from there? As stated at the top of the page, we don't do your homework for you. Dismas|(talk) 21:08, 2 October 2005 (UTC)[reply]

Indian Economic Planning

What are the features of Indian Economic Planning? What are the objectives of planning in India? What suggestions would you make to have better plans?

See the answer directly above this question. Dismas|(talk) 21:13, 2 October 2005 (UTC)[reply]

nutrition

What are the nutritional values of peppermint tea?

None, in a strict sense. There's zero calories and zero other "nutrients", as such. However, there are trace amounts of various chemicals, such as caffiene, which have some effect on the body. Of course, if you add milk or sugar, they retain their nutritional content when placed in the tea. --Robert Merkel 04:11, 3 October 2005 (UTC)[reply]

Scanners

is a scanner a input or output device

Input, for more info, please see the article on Image scanners. Dismas|(talk) 21:16, 2 October 2005 (UTC)[reply]

Mexico's imports

Try Economy of Mexico --Borbrav 22:37, 2 October 2005 (UTC)[reply]

death on your birthday

What is the term for someone who dies on the same date they were born, only different year? Deceased.

Ha! Copying this to BJAODN :)--inksT 22:41, 2 October 2005 (UTC)[reply]

What region Kentucky is in

Is Kentucky in the South or the Midwest?

Yes. It's sometimes classed as Midwestern, and it's sometimes classed as Southern. The culture is more Southern, and I think that's where most people conceptualize it. - Nunh-huh 22:16, 2 October 2005 (UTC)[reply]

It had trouble making up its mind in the Civil War too. Most midwesterners think of it as more southern culturally, and I suspect so most kentuckians, but the division is more a cultural gradient than a clear boundary. alteripse 22:34, 2 October 2005 (UTC)[reply]

Year the late 1980s began

What year did the late 1980s begin?

A phrase like that is used precisely to avoid questions like yours. This is not a snotty answer but an accurate one. If the user of the phrase could have fixed an exact boundary time, he probably wouldn't have chosen this imprecise phrase. To me late 80s suggests 85, 86, or 87, running to 88, 89, or 90 but the edges are fuzzy. I am guessing you either are desperately trying to salvage a losing argument, have Asperger syndrome, or are King Nitpicker. Want to share? alteripse 00:34, 3 October 2005 (UTC)[reply]

Who sang the song in the movie, itself?

WHO actually sang the song 'Three Coins in the Fountain' in the movie "Three Coins in the Fountain"?

DVD boxes

For what reasons, other than the fact that people might get confused, are DVD cases not the same size and shape of CD cases

what is the fastest a man can run in miles per hour

Your question is a bit vague. The fastest average speed for a race (at any distance) recorded is Michael Johnson's 200 metre world record, which he completed in 19.32 seconds. If you do the maths, that works out to a speed of 23.23 mph. However, his peak speed was obviously somewhat higher because he had to accelerate from rest. Over longer distances, of course, the average speed that a human can maintain is considerably reduced. The current world marathon record, at 2 hours, 4 minutes, and 55 seconds for a 42.195 kilometre course, gives an average speed of 12.52 mph. Average people are obviously considerably slower than these world record times. --Robert Merkel 04:06, 3 October 2005 (UTC)[reply]

Credit Cards

What do Credit Cards companies do to an outstanding balance of a dead individual? Does death need to be proven?

Such a balance would probably be treated like any other debt; i.e., it would be paid out of the estate during probate. Your local law may differ. You should probably ask an estate lawyer; http://lawyers.com may help you find one. —Charles P. (Mirv) 02:48, 3 October 2005 (UTC)[reply]

Who is "george giri"

George Giri is a lawyer and a law teacher, who is also a human rights activist in India. He is well versed in international law and Corporate law. He is the author of many law books such as "Company Law", "Science, tehnology and Human Rights", "Human Rights of Women prisoners", "International Law- A Text book", "Intellectual Property law", "Patentability of Biotechnological innovations", etc. He graduated in English Literature from University of Madras and has done his law degree from V.M.Salgaocar College of Law,Goa. He has completed his LL.M degree from a prestigious law school of India, namely, the HIDAYATULLAH NATIONAL LAW UNIVERSITY at Raipur.